SlideShare una empresa de Scribd logo
1 de 144
AIR BORNE DISEASES
& CASES APPLICATIONS
By
Doaa Habib
Under supervision of
Prof dr Mona Aboserea
Zagazig university
CONTENT
 Infectious chain
 Mode of transmission
 Definitions
 Classification
 Viral infections
 Emerging airborne infections
 Bacterial infections
 Questions
INFECTIOUS CHAIN
 Air borne or droplet or respiratory tract infections:
 Agent : virus, bacteria, fungi.
 Reservoir: mainly man, animal.
 Exit: respiratory secretion.
 Mode of transmission: inhalation
 Inlet: nose, mouth.
 Host: susceptible individual.
MODE OF TRANSMISSION
 Direct
Droplet spread
 Indirect
Airborne
Indirect contact
DEFINITIONS
 Air borne transmission :
 occurs when infectious agents are carried by dust
or droplet nuclei suspended in air.
 Droplet nuclei are dried residue of less than 5
microns in size.
 Droplet nuclei may remain suspended in the air for
long periods of time and may be blown over great
distances.
 for example, Measles and TB .
DEFINITIONS
 Droplet spread :
 Refers to spray with relatively large, short-range
aerosols produced by sneezing, coughing, or even
talking.
 Droplet spread is classified as direct because
transmission is by direct spray over a few feet,
before the droplets fall to the ground.
 Examples, Pertussis and meningococcal infection.
CLASSIFICATION
CLASSIFICATION
Less common infections:
Viral: herpes simplex, Epstein-bar, hand, foot and mouth
disease.
Bacterial: chlamydia pneumonia, legionella, mycoplasma
Fungal: Candida , Coccidioidomycosis, Cryptococcosis,
Histoplasmosis, Pneumocystis carinii pneumonia.
VIRAL: INFECTIOUS MONONUCLEOSIS AND HERPES
SIMPLEX
 Caused by Epstein Bar virus in case of infectious
mononucleosis and Herpes simplex virus in case of
Herpes simplex.
 Infectious Mononucleosis is manifested by sore
throat, lymphadenopathy, splenomegaly, skin rash
and stomatitis.
 Herpes Simplex usually produce vesicular
stomatitis. The illness may starts acutely with fever
and oral pain, followed by appearance of lesions on
lips, gums, mucosa, tongue, palate.
Management
 Symptomatic
 IV fluid in case of severe difficulty in swallowing.
 Gentian violet helps to prevent secondary infection in Herpes
simplex.
 If secondary bacterial infection occurs, penicillin or
metronidazole should be prescribed
VIRAL: HAND, FOOT, AND MOUTH DISEASE
 Hand, Foot, and Mouth Disease, or HFMD, is a viral
contagious illness caused by coxsackie virus common in
infants and children younger than 5 years old.
Transmission: Close contact, droplet, contact with blister fluid
 c/p: fever, mouth sores, and a rash, HFMD is usually not
serious, and nearly all people recover in 7 to 10 days
without medical treatment.
 Complication: Rarely, viral meningitis polio-like paralysis, or
encephalitis HFMD spreads from an infected person to
others from:
Management: symptomatic
BACTERIAL: CHLAMYDIA PNEUMONIA
 Agent: C. pneumoniae
 IP: 3 to 4 weeks
 Transmission: close contact , respiratory droplets, contact with
contaminated surfaces.
 c/p: cough, fever, headache, malaise and can develop:
 Laryngitis, Pharyngitis, Pneumonia
 Complication: Exacerbation of asthma, Encephalitis, Myocarditis
 Diagnosis:
1. culture
2. Serology: CF, Micro immunofluorescence (MIF) is the serological
method of choice
3. PCR
 Management:
1) Macrolides (azithromycin) — first-line therapy
2) Tetracyclines (tetracycline and doxycycline): not for children
3) Fluoroquionolones
LEGIONNAIRES’ DISEASE
 Agent: Legionella bacteria (L. pneumophila)
 Reservoir: natural, freshwater environments, human-made water systems
(e.g., plumbing system of large buildings, cooling towers, decorative
fountains, hot tubs)
 IP: 3 to 4 weeks
 Transmission: inhaling aerosolized water droplets, aspiration of
contaminated drinking water. Legionella is usually not transmitted from
person to person. however, a single episode of person-to person
 Risk Factors
 Age ≥50 years
 Smoking (current or historical)
 Chronic lung disease, such as emphysema or COPD
 Immune system disorders due to disease or medication
 Systemic malignancy
 Underlying illness, such as diabetes, renal failure, or hepatic failure
Legionnaires’ disease
Pontiac fever
Pontiac fever
Clinical features Fever, myalgia, and cough
shortness of breath, headache, confusion,
nausea, diarrhea may be present
flu-like illness, often with fever,
chills, headache, myalgia,
fatigue, malaise; less often with
symptoms such as cough or
nausea
Pneumonia
(clinical or
radiographic)
Yes No
Pathogenesis Replication of organism Possibly an inflammatory
response to endotoxin
Incubation
period
2 to 10** days after exposure 24 to 72 hours after exposure
occuracce Less than 5% of people exposed to the
source of Legionella
Greater than 90%
3
Treatment Antibiotics Supportive care (because illness
is self-limited)
Isolation of the
organism
Possible Never demonstrated
Outcome Hospitalization common
Case-fatality rate: 10% (25% for
healthcare-associated)
Hospitalization uncommon
Case fatality rate: extremely low
LEGIONNAIRE DISEASE
 Diagnosis: Best practice is to obtain both sputum culture and
the urinary antigen test
Indications for Legionnaires’ Disease Testing:
 Patients who have failed outpatient antibiotic treatment
for community-acquired pneumonia
 Patients with severe pneumonia, in particular those
requiring intensive care
 Immunocompromised patients with pneumonia*
 Patients with a travel history (patients who have traveled
away from their home within 10 days before the onset of
illness)
 All patients with pneumonia in the setting of a
Legionnaires’ disease outbreak
 Patients at risk for Legionnaires’ disease with
healthcare-associated pneumonia (pneumonia with
onset ≥ 481 hours after admission)
 Management: Macrolides and respiratory
fluoroquinolones
MYCOPLASMA PNEUMONIA
 Agent: M. pneumoniae
 IP: 1 to 4 weeks
 Transmission: airborne droplets from person to person and is exclusively
a human pathogen.
 c/p: cough, fever, headache, malaise and can develop: Tracheobronchitis,
Pharyngitis Pneumonia
 Complication: Exacerbation of asthma, Encephalitis, Myocarditis,
Hemolytic anemia, Renal dysfunction, Erythema multiforme, Stevens-
Johnson syndrome, or toxic epidermal necrolysis
 Diagnosis:
1. culture
2. Serology: enzyme immunoassay (EIA) testing
3. PCR
 Management: it does not respond to sulfonamides or penicillin (beta-
lactams).
1) Macrolides (azithromycin) — first-line therapy
2) Tetracyclines (tetracycline and doxycycline): not for children
3) Fluoroquionolones
FUNGAL INFECTION
Candidiasis This illness is caused by Candida. Candidiasis, can affect the skin, nails, and
mucous membranes throughout the body.
Persons with HIV infection often have trouble with Candida, especially in the
mouth and vagina. However, candidiasis is only considered an OI when it
infects the esophagus (swallowing tube) or lower respiratory tract, such as the
trachea and bronchi (breathing tube), or deeper lung tissue.
Coccidioidom
ycosis
This illness is caused by the fungus Coccidioides immitis.
It most commonly acquired by inhaling fungal spores, which can lead to a
pneumonia that is sometimes called desert fever, San Joaquin Valley fever, or
valley fever.
Cryptococcos
is
This illness is caused by infection with the fungus Cryptococcus neoformans.
The fungus typically enters the body through the lungs and can cause
pneumonia.
Histoplasmosi
s
This illness is caused by the fungus Histoplasma capsulatum.
Histoplasma most often infects the lungs and produces symptoms that are
similar to those of influenza or pneumonia. People with severely damaged
immune systems can get a very serious form of the disease called progressive
disseminated histoplasmosis.
PNEUMOCYSTIS PNEUMONIA
 Agent: fungus Pneumocystis jirovecii (carinii)
 IP: 1 to 4 weeks
 Risk: HIV, Solid organ transplant, Blood cancer, Inflammatory diseases or
autoimmune diseases, Stem cell transplant
 Transmission: airborne droplets from case or carrier.
 c/p: cough, fever, headache, Difficulty breathing, Chest pain, Chills
Pneumonia
 Complication: highly fatal.
 Diagnosis:
1. sputum or bronchoalveolar lavage Or lung tissue biopsy is examination
under a microscope.
2. PCR
3. A blood test to detect β-D-glucan
 Management:
trimethoprim/sulfamethoxazole also known as co-trimoxazole
 Prevention: co-trimoxazole for patients at risk.
German measles
“Rubella”
Measles
“Rubeola”
Cases & incubatory
Carriers. Congenitally
Infected infants act as
reservoir for 1 year"in
respiratory secretion, blood,
urine, & stool”
Man only case or contact
healthy carrier (inresp.
secretion)
Source of
infection
14-18 days10 daysIP
1 week before & 7 days
after rash
In prodromal stage & 5
days after rash
Period of
infectivity
German measles
“Rubella”
Measles
“Rubeola”
C/P
- 1 day
- Mild symptom
- Cervical lymph adenopathy.
- 3-4 days
Fever, cough, Catarrh, Conjunctivitis
- Koplik's spots on 2nd day “Tiny
bluish white spots on buccal mucosa”.
Prodroma
lstage
- Lasts for 3 days.
- Maculo-papular or uniform red all
over body.
On 4th day red blotchy appears on
face, root of hair then generalized.
Branny desquamation after 1 week.
Rash
- Congenital rubella syndrome
“CRS”: infection during 1st
trimester(16 weeks). Causes
cataract, deafness, heart
anomaly, mental retardation.
- Otitis media, cervical adenitis
- Encephalitis
- Pneumonia
- Otitis media
Complication
German measles
“Rubella”
Measles
“Rubeola”
Pre
vent
ion
Rubella alone or MMR live
attenuated, single dose 0.5ml
SC. given at 12-15th month.
Given to adolescent girls or
females at any age before
pregnancy.
MMRV:12M - 12 years of age.
The minimum interval is 3
months
Life long immunity.
Type: live attenuated virus
Preparations: MMR, MMRV
Administration: 0.5 ML subcutaneous
injection.
Schedule:
MMR: 2 doses, 1st at 12-15 months and
booster dose( at least 28 day after 1st dose)
at 4-6 years before school entry.
Life long immunity
Vaccination
Vaccine and IGS are not effective
Pregnant exposed to infection &
refuse abortion.
Vaccine within 3days of exposure is protective
Within 6 days of exposure
Sero-prevention: Human Ig.
Sero-attenuation: smaller dose to give mild C/P.
PEP
ZosterChicken poxSmall pox
Preparation, type:
Zostavax: live
attenuated virus.
Shingrix
:recombinant zoster
vaccine
Administration: 0.5ml
SC injection in arm
Schedule:
Zostavax: single
dose.
Shingrix : IM two
doses separated by
2 to 6months.
Type: live attenuated
virus
Preparation: varivax
(varicella alone), MMRV
Administration: 0.5 ML
subcutaneous injection
Schedule:
First dose at 12 -15
months old, Second dose
at 4 -6 years
Calf lymph vaccine: vaccinia
virus
ACAM2000
Aventis Pasteur Smallpox
Vaccine (APSV):
Imvamune: attenuated live
vaccine, underdevelopment.
Administration, Schedule :
Calf lymph vaccine: single
dose by scratching.
ACAM2000, Aventis Pasteur
Smallpox Vaccine (APSV):
single dose by the
percutaneous route using the
multiple puncture technique.
Vaccination
varicella vaccine: within 3 daysup to 5 days, of
exposure to rash.
zoster immune globulin (ZIG): within 72 to 96
hours after exposure
Vaccination Within 3 days
prevent, 5-7 days mild
symptoms
PEP
VIRAL INFECTIONS WITHOUT RASH
MumpsCommon coldinfluenza
VirusRhino v.Virus types
A:mutation
&pandemics
B:epidemic
C:outbreaks
Agent
Cases &
incubatory
carriers (saliva
& in utero)
CasesHuman cases
only “may be
Avian, swine,
horses”
Source of
infection
18 days1-3 daysIP
1 week before
parotitis till
disappearance
of swelling.
All course of diseasePeriod of
infectivity
MumpsCommon
cold
influenza
Prodroma: 1-2 days
fever, malaise,
headache, body aches.
Enlarged painful
salivary gland (parotid,
sublingual,
submaxillary)
Sudden
onset
catarrh,
running
nose,
sneezing,
sore throat,
cough.
Usually no
fever
Sudden high
fever, body
aches, arthralgia,
sore throat,
cough.
C/P
Orchitis, oophritis,
aseptic meningitis,
pancreatitis, mastitis,
nephritis, neuritis
Rare.Pneumonia, otitis
media, sinusitis,
pericarditis,
bronchitis
Complication
Mumps C.C influenzaVaccination
MMR 0.5ml SC or
mumps vaccine alone.
Must be before
puberty.
Life long immunity.
No 1- Inactivated :
Trivalent: 5 years – 65 years, high dose,
adjuvant vaccine > 65 years
Quadrivalent: 6months – 64 years
2- Live attenuated:
Trivalent, Quadrivalent: 2 years – 64 years
3- Quadrivalent Recombinant: > 18 years
Administration, Schedule :
Inactivated:0.5 ml IM, 2 doses separated
by 4 weeks.
Live attenuated: 0.2 ml intranasal, one
dose.
PEP
booster dose of MMR
within 5 days
Chemoprophylaxis: Oseltamivir,
Zanamivir
EMERGING VIRAL AIR BORNE INFECTIONS
 Avian flu( H5N1)
 Swine flu (H1N1)
 SARS
 MERS- COV
Swine fluAvian flu
H1N1H5N1, H7N9(Asian form)
Ag
en
t pigsPoultry, wild birds, pigs
S.
INF
2 - 14 days
(internationally 7 days)
14 days (internationally 7 days)
IP
All coarse of diseaseMan to man transmission Is very rare
infec
tivity
mild to severe and included conjunctivitis, influenza-like illness (e.g., fever,
cough, sore throat, muscle aches), pneumonia sometimes accompanied by
nausea, abdominal pain, diarrhea, and vomiting
C/P
Fatality is lowacute respiratory distress, respiratory failure),
neurologic changes (altered mental status,
seizures), and the involvement of other organ
systems.
Complicati
on
The H1N1 Inactivated
Vaccine (2 dose, IM)
The H1N1 Live
Attenuated Intranasal
Vaccine
Vaccine for birds
Vaccination
Like flu
P
E
P
CLASSIFICATION
Scarlet feverStrpt.
Pharyngi
tis
Diphteria
NoUnder
trial
Type:
Diphtheria: toxoid
Preparation:
1- Diphtheria and Tetanus (DT and Td)
2- Diphtheria, Tetanus, acellular
Pertussis (DTaP) Vaccines
3- Tetanus, Diphtheria, acellular
Pertussis (Tdap) Vaccines
IM injection on 2, 4, 6 months, 15 -18
months, and 4 through 6 years
Vaccination
Chemoprophylaxis is not
recommended
Antibiotics:
benzathine penicillin G, erythromycin.
Diphtheria antitoxin is not indicated for
prophylaxis of contacts of diphtheria
patients.
PEP
TBPertussisMeningitis
Preparation, type:
Live attenuated bovine
strain.
Administration,
Schedule :
0.1 ml intradermal
injection in deltoid of lt
arm, single dose
Type:
Pertussis; killed
bacteria in DPT
Preparation:
1- Diphtheria and
Tetanus (DT and Td)
2- Diphtheria,
Tetanus, acellular
Pertussis (DTaP)
Vaccines
3- Tetanus,
Diphtheria, acellular
Pertussis (Tdap)
Vaccines
IM injection on 2, 4,
6 months, 15 -18
months, and 4
through 6 years
Capsular polysaccharide vaccine:
quadrivalent, capsular polysaccharide
Meningococcal conjugate: quadrivalent,
meningococcal A, C, W, and Y
polysaccharides conjugated to protein
adjuvant.
serogroup B meningococcal vaccines:
monovalent, recombinant protein
vaccines
Administration, Schedule :
Capsular polysaccharide vaccine: 0.5
ml IM, 2 doses 3months apart
Meningococcal conjugate: 0.5 ml IM, 2
doses 2 months apart
serogroup B meningococcal
vaccines:0.5 ml IM, 2 doses 1months
apart
Vaccination
INH orally for 1 year .Oral erythromycin in
4 divided doses for
10 days
Rifampin 600mg twice 2 days
International certificate to endemic
area, ciprofloxacin, and ceftriaxone
may be used
PEP
DPT VACCINE
Preparation:
1- for children < 7 years (DTaP and DT)
2- for persons > 7 years (Tdap and Td)
They differ in concentrations of diphtheria and tetanus concentrations
1- for children < 7 years: concentration of DT toxoid (25 Lf of diphtheria toxoid,
10 Lf of tetanus toxoid) is higher than td and D toxoid concentration is higher
than T toxoid.
2- for persons > 7 years: concentration of td toxoid (5 Lf tetanus toxoid, 2 Lf
diphtheria toxoid) is lower than TD and t toxoid concentration is higher than d
toxoid.
Contraindications to acellular pertussis-containing vaccines:
1. Patients who developed encephalopathy within 7 days of administration of
a previous dose of DTP, DTaP, or Tdap
2. Progressive or unstable neurologic disorder (including infantile spasms for
DTaP)
3. Uncontrolled seizures
4. Progressive encephalopathy
5. Had Guillain-Barré syndrome within 6 weeks after a previous dose of
tetanus toxoid-containing vaccine
Pneumococcal diseaseHib disease
lancet-shaped, gram-positive capsulatedgram-negative coccobacillus.
Mostly encapsulated
Ag
nasopharyngeal carrier or case patientnasopharyngeal carrier or case patient.
aspiration of amniotic fluid or contact
with genital tract secretions
S.inf
1 to 3 days2-4 days
Ip
transmission can occur as long as the organism
appears in respiratory secretions.
Secondary attack rate is 2-6%
p.If
Ear infections, Sinus infections, Pneumonia,
Meningitis, Bacteremia
Pneumonia, Bacteremia, Meningitis,
Epiglotittis, Cellulitis, Infectious arthritis
C/P
Preparation, type:
pneumococcal conjugate vaccine (PCV7): IM
Pneumococcal Conjugate Vaccine (PCV13);
Pneumococcal Polysaccharide Vaccine
(PPSV23): IM or SC Administration, Schedule:
(2 doses, 8 weeks apart)
Routine: at 2, 4, and 6 months of age, Booster
dose at 12 through 15 months of age.
Preparation, type: a polysaccharide
conjugate vaccine.
Monovalent: Hib
Combined: pentacel ( DTaP + Hib + IPV)
Administration, Schedule:
IM injection at 2,4,6 months OR 2,4
months
and booster 12 - 15 months
Vacc
Not recommendedRifampicin 600mg twice daily
pep
CASE STUDY
A "5" years old girl was bought to the outpatient clinic because
of rash covering the girl's face, it was preceded by fever,
malaise for 3 days.
a)What is the differential diagnosis?
On examination, the rash was red blotchy maculopapular rash
and there was bluish spot on the girl’s buccal mucosa.
b) What is the provisional diagnosis?
c) How to confirm?
d)What are the preventive and control measures that should be
done for the case, house hold contact including 10 months
brother and her school contacts .
CASE 1
a) Differential diagnosis:
 Measles (maculopapular, first on face, on 4th day of high
fever)
 Rubella (maculopapular, after 1 day of low fever , cervical
lymph nodes)
 Varicella ( pleomorphic, after 2-3 days of low fever, first on
trunk centripetal)
 smallpox ( monomorphic, high fever, first on extremities)
Less likely
 Scarlet fever (no rash in the face)
 Meningitis (high fever, convulsions)
 Drug or food allergy (history of exposure)
CASE 1
b) the provisional diagnosis: probable case of measles
c) Confirm diagnosis:
 Isolation of measles virus from a clinical specimen; or
 Detection of measles-virus specific nucleic acid from a clinical
specimen using polymerase chain reaction; or
 IgG seroconversion or a significant rise in measles
immunoglobulin G antibody using any evaluated and validated
method; or
 A positive serologic test for measles immunoglobulin M antibody;
or
 Direct epidemiologic linkage to a case confirmed by one of the
methods above.
CASE 1
Case: notification to LHU- isolation (4 days after rash in measles)- concurrent
disinfection-ttt- release after rash disappearance.
Treatment
 There is no specific antiviral therapy. Symptomatic, address complications such as
bacterial infections.
 Severe measles cases among children should be treated with vitamin A.
Contact: enlistment- surveillance - HE
Segregation: from school, child care, health care setting
MMR vaccine: within 72 hours of initial measles exposure to those without evidence
of immunity
Immunoglobulin (IG) :
People who are at risk for severe illness and complications from measles, such as
infants younger than 12 months of age, pregnant women and immuno compromised
people
Its recommended to give vaccine rather than IGS to the baby and later on he take
the routine 2 doses of MMR
School contacts
If sure of immunization (document 2 doses): survillance- HE to avoid further
exposure.
If not sure of previous immunization, or child have fever or respiratory catarrah
before or the child is immunocompromised: seroprevention, seroattenuation
Outbreak measures: booster dose MMR
CASE 2
A student 7 years old complaining of fever, sore throat,
headache and malaise was detected by school nurse.
a)What is the provisional diagnosis?
b)What is the differential diagnosis?
c)What are the preventive measures that should be done in
school according to the type of disease identified?
d)What are the measures taken for the sick child? His house
hold contacts?
CASE 2
 provisional diagnosis: Streptococcal pharyngitis.
 differential diagnosis:
 Strept. Pharyngitis
 Diphtheria
 Influenza :short I.P(1-3 days), sudden onset of fever,
constitutional manifestation
 Common cold: mild fever, running nose
 Less likely: Meningitis: high fever, convulsions
 Prodroma of measles, rubella
CASE 2
preventive Measures done in school
General preventive measures: good ventilation, HE, notification
if get ill
In strept: chemoprophylaxis is not recommended
In diphtheria: close survillence and administer antitoxin with the
first sign of disease.
 if immunized before: booster dose of D toxoid, not
immunized 2 doses of vaccine.
 prophylaxis by antibiotics to all contacts
 benzathine penicillin G Single dose. or
 a 7- to 10-day course of oral erythromycin (40 mg/kg/day for
children and 1 g/day for adults).
CASE 2
measures taken for the sick child? His house contact ?
Child:
Penicillin or amoxicillin is the antibiotic of choice to treat group A strep pharyngitis. for 10
days
Diphtheria:
 Respiratory support and airway maintenance should also be administered as needed.
Diphtheria Antitoxin
 Diphtheria antitoxin, 20,000 - 80,000 I.U. (or even 100,000 in severe cases) I.M. or I.V.
 The patient must be tested for sensitivity before antitoxin is given.
Antibiotics:
The antibiotics of choice are
 erythromycin (500 mg four times daily for 14 days) or
 procaine penicillin G (600,000 units every 12 hours intramuscularly) until the patient can
take oral medicine, followed by oral penicillin V (250 mg four times daily) for a total
treatment course of 14 days.
 Elimination of the organism should be documented by two consecutive negative cultures
after therapy is completed at least 24 hours apart .
 Maintain isolation until elimination of the organism .
 Vaccination is required because clinical diphtheria does not necessarily confer immunity
CASE 2
House contacts:
Strept. Pharyngitis: HE, SURVIELLENCE FOR 3 DAYS
Diphtheria:
 close surveillance and begin antitoxin at the first signs of illness.
 a diphtheria booster if vaccinated before, 2 doses if not vaccinated.
AND
 Antibiotics:
 benzathine penicillin G Single dose (600,000 units for persons younger
than 6 years old and 1,200,000 units for those 6 years old and older) or
 a 7- to 10-day course of oral erythromycin (40 mg/kg/day for children and
1 g/day for adults).
 Identified carriers should also receive antibiotics.
 Adult with occupational contact with children —» allow to go to work only
after 2 more -ve swabs.
 If the swab is positive —» Segregate (exclude) from school or work until 3
consecutive negative swabs 24 hours apart is obtained.
 Diphtheria antitoxin is not indicated for prophylaxis of contacts of diphtheria
patients
CASE 3
A male adult aged 50 years old working as public bus driver,
complaining of low fever, loss of weight, night sweating, cough
and anorexia. Symptoms began since 2 weeks.
a) What is the provisional diagnosis?
b) How can you verify your diagnosis?
c) What are the preventive measures that must be done for that
case, wife, 1 and 18 years old children and his old father 70
years?
CASE 3
the provisional diagnosis:
Probable Pulmonary tuberculosis
- history: elderly-male patient
- c/p: low grade fever, loss of wt, anorexia, night sweating, cough
Confirm diagnosis:
A) Bacteriology:
1) Detection of TB bacilli ( Direct smear microscopy, laryngeal
swabs, culture).
2) Tuberculin test ( immune response to TB).
3) Histo-pathological diagnosis.
4) BACTEC ( detection of metabolic end products of bacilli).
5) PCR ( detection of DNA of TB bacilli).
B) Radiology: sensitive but not specific
CASE 3
Measures for case: case finding, notification to LHU, isolation for 4-8 ws,
disinfection, ttt, follow 5 years
Treatment: DOTS
 first 2 months: isoniazid + rifampicin + pyrazinamide + ethambutol
 following 4 months: isoniazid + rifampicin
Measures for Contacts: enlistment- HE
 tuberculin testing
 tuberculin -ve give BCG vaccine and prevent contact with the case till 3
months
 tuberculin +ve do chest x ray, if suspicious do smear exam. Give INH.
Chemoprophylaxis: INH (isonicotinic acid hydrazid) orally for 1 year.
Year: BCG in first 3 months, so TST is +ve > 15mm
Sputum exam (+ve: DOTS), (-Ve: chemoprophylaxis)
18 years: TST test, sputum exam
If both +ve (DOTS)- IF both –ve ( BCG)
IF TB +ve and sputum –ve: chemoprophylaxis
Father (70 ys), Pregnant: immunocompromised
C.I of BCG>35ys, INH risk of induce hepatitis
CASE 4
A 15 years old girl came to the out patient clinic complaining
from high fever, vomiting, neck rigidity and convulsion.
a) What is the differential diagnosis?
On examination there was dark red petechial eruption on
extremities.
b) What is the provisional diagnosis?
c) How to confirm?
d) What is the measures taken to the case and her contacts?
CASE 4
a) Meningitis for differential diagnosis
Bacterial:
 Neisseria meningitides
 haemophilus influenza
 Streptococcus pneumoniae
 Group B Streptococcus
 Listeria monocytogenes
Viral:
 Non-polio enteroviruses, the most common
 Mumps virus
 Herpesviruses, including , herpes simplex viruses, and varicella-
zoster virus.
 Measles virus
 Influenza virus
 Arboviruses, such as West Nile virus
Fungal: in immunocompromised Cryptococcus, Histoplasma
Non infectious: autoimmune, injury, tumors
CASE 4
CASE 4
b) Provisional diagnosis: suspected meningococcal meningitis
due to presence of purpura fulminans.
c) Confirm diagnosis:
 Detection of N. meningitidis-specific nucleic acid in blood or
CSF), using a valudated polymerase chain reaction (PCR)
assay; or
 Isolation of N. meningitidis From blood or CSF, or less
commonly or purpuric lesions.
CASE 4
d) Measures to case, contact.
Case:
 effective antibiotics should be administered promptly to patients
suspected of having meningococcal disease.
 treatment with penicillin G, ampicillin, or an extended-spectrum
cephalosporin (cefotaxime or ceftriaxone) .
 if antimicrobial agents other than ceftriaxone or cefotaxime are
used for treatment, eradication of nasopharyngeal carriage with
rifampin (4 doses over 2 days) or single doses of ciprofloxacin or
ceftriaxone are recommended prior to discharge from the hospital.
Contact:
 Antibiotic chemoprophylaxis: ideally should be initiated within 24
hours after the index patient is identified; prophylaxis given >2
weeks after exposure has little value.
 Antibiotics used ciprofloxacin, rifampin, and ceftriaxone.
 Ceftriaxone is recommended for pregnant women.
 Rifampin 600mg twice daily.
CASE 5
A 35 years old man come to out patient clinic complaining from
chest pain, productive Cough, Dyspnea and high fever
The patient is smoker for 5 years.
a) What is provisional diagnosis?
b) What is the causative agent of the disease?
c) The gram stain shows gram positive cocci, How to confirm
diagnosis?
d) What are measures taken for the patient and his contacts?
CASE 5
a) provisional diagnosis: pneumonia
b) Causative organism:
Viral: influenza and respiratory syncytial virus (RSV)
Bacterial: pneumococcus, legionella, mycoplasma, H. influenza
Fungal: pneumocystis pneumonia.
c) pneumococcal pneumonia confirmed by:
 Supportive: Identification of S. pneumonia from a normally sterile
body site by a CIDT(culture independent diagnostic tests like PCR
) without isolation of the bacteria.
 Detect capsular polysaccharide antigen in body fluids or in urine
by immunochromatographic membrane technique.
 Confirmatory: Isolation of S. pneumonia from a normally sterile
body site by culture.
CASE 5
d) Measures taken for the patient and his contacts:
Patient: notification, isolation, disinfection, treatment
General: support respiration, symptomatic.
Specific: S. pneumonia are resistant to one or more clinically relevant
antibiotics so treatment depend on culuture and sensitivity.
Contacts: enlistment, HE, surveillance for 3 days
Chemoprophylaxis
The American Academy of Pediatrics typically recommends daily
antimicrobial prophylaxis with oral penicillin V or G for children with
functional or anatomic asplenia, especially those with sickle-cell disease.
Because secondary cases of invasive pneumococcal infection are
uncommon, chemoprophylaxis is not indicated for contacts of patients with
such infection.
Vaccination: pneumococcal conjugate vaccine (PCV13), pneumococcal
polysaccharide vaccine (PPSV23)
 Are recommended for all children, old age, adults with certain diseases
 It is also recommended after infection because infection doesnot prevent
future attacks.
CASE 6
5 years girl was brought to your office by her mother
complaining from pruritic vesicular rash covering her body and
preceded by fever.
a) What is your diagnosis?
b) How to confirm it?
c) What are the measures taken for the girl and her contacts?
CASE 6
a) Diagnosis: probable case of chicken pox (varicella infection)
b) Confirm diagnosis:
 Epidemiologic linkage to another probable or confirmed
case, OR
 Laboratory confirmation by any of the following:
 Isolation of varicella virus from a clinical specimen, OR
 Varicella antigen detected by direct fluorescent antibody
test, OR
 Varicella-specific nucleic acid detected by polymerase chain
reaction (PCR), OR
 Significant rise in serum anti-varicella immunoglobulin G
(IgG) antibody level by any standard serologic assay
CASE 6
c) Measures to girl & contacts :
The girl: notification, isolation 7 days, disinfection ,ttt
 symptomatic, analgesics, calamine lotion and antihistaminic
 treatment with antivirals is not routinely recommended for
otherwise healthy children with varicella.
Contacts:
Previous Varicella infection or 2 doses of vaccine give life long immunity, no
need to PEP.
varicella vaccine: is effective in preventing illness or modifying varicella
severity if administered to unvaccinated children within 3 days, and possibly
up to 5 days, of exposure to rash.
zoster immune globulin (ZIG): within 72 hours of exposure , 96 hours after
exposure in Immuno compromised patients but may be effective if
administered as late as 10 days .
CASE 7
11 years old boy come to your office complaining from fever
and swelling behind his ear.
a) What is the possible diagnosis?
b) What are the complications of such case?
c) What are the measures taken for the boy and his contacts?
CASE 7
a) Diagnosis: suspected mumps case
b) Complication :
 Orchitis, usually unilateral in 20-30% of post pubertal males
(rarely may induce sterility)
 CNS involvement (aseptic meningitis, Encephalitis).
 Pancreatitis.
 Neuritis, arthritis, mastitis, nephritis, thyroiditis and
pericarditis may occur.
 Permanent nerve deafness (usually unilateral) is a rare
complication.
CASE 7
c) Measures for the boy & contacts:
The boy: notification, isolation, disinfection, ttt.
Isolation: relieve of swelling (9 days).
Treatment: symptomatic
Contacts: enlistment, segregation, HE, surveillance for 2
weeks.
 2nd dose of MMR within 5 days.
 third MMR dose administered as PEP did not have a
significant effect, it may offer some benefits in specific
outbreak contexts.
 Immune globulin (IG) is not effective postexposure
prophylaxis.
CASE 8
10 months baby boy brought by his mother to your office
complaining from attacks of productive cough which increase at
night and end with vomiting. The mother also complained from
strange cough sound.
a) What is the provisional diagnosis?
b) What are the complication of this disease?
c) What are the measures taken to the baby and his contacts?
CASE 8
a) Diagnosis: whooping cough ( pertussis infection).
b) Complications:
1- Increasing pressure during paroxysmal coughing:
* Hernia (especially umbilical) and prolapse of rectum.
* Convulsions in infants due to cerebral anoxia.
* Hemorrhage: skin, nasal, conjunctiva and C.N.S.
*Encephalopathy in severe cases from cerebral anoxia or hemorrhage.
2- Secondary bacterial infection: may cause otitis media, diarrheal disease,
pneumonia and bronchopneumonia.
3- Malnutrition: due to repeated vomiting for long time in untreated cases.
4- Long term complications which include neurological deficits ranging from
gross mental retardation to behavior disorders.
CASE 8
c) Measures for baby & contact:
Baby: Case finding, Notification, Isolation , disinfection.
Treatment:
 Specific:
- erythromycin : administered in 4 divided daily doses for 14 days
 symptomatic:
- Proper feeding.
- Prevention and management of complications.
- Release: pupils can return to school after 3 weeks from start of whooping
stage and improvement of the case clinically or one week from beginning of
effective treatment.
Contacts: enlistment, HE, surveillance 2 weeks
Chemoprophylaxis: Oral erythromycin 50 mg/kg/day in 4 divided
doses for 10 days.
Providing PEP to all household contacts of a pertussis case.
CASE 9
65 years old man visit the out patient clinic complaining from
painful rash on his shoulder for 3 days?
a) What is the provisional diagnosis?
b) What is the complication?
c) What are the measures taken for the patient and his
contacts, wife, pregnant daughter , and 10 years grandson?
CASE 9
a) Diagnosis: zoster infection (VZV )
b) Complications:
 post-herpetic neuralgia
 cranial or peripheral nerve palsies
 sensory loss, deafness
 ocular complications.
 bacterial infection of the lesions, usually due to
Staphylococcus aureus.
 visceral involvement, meningoencephalitis, pneumonitis,
hepatitis, and acute retinal necrosis.
CASE 9
c) Patient: notification, isolation, disinfection
 Antiviral drugs: acyclovir, valacyclovir, and famciclovir
 Analgesics
 Wet compresses, calamine lotion to relieve some of the
itching.
 Vaccination (Shingrix ) after acute attack to prevent coming
episodes.
Contacts: enlistment, HE, surveillance
Wife:
 Evidence of varicella immunity: nothing
 No Evidence of immunity and immunocompromised: VZIG
during 72 – 96 Hours OR acyclovir for 7 days after 7 -10 days post exposure.
 Shingrix should be offered
CASE 9
Pregnant daughter:
 Evidence of varicella immunity: nothing
 No Evidence of immunity:
 VZIG during 72 – 96 Hours.
 if she has signs and symptoms of varicella around the time of delivery 5
days before to 2 days after delivery, VZIG should be given to her neonate.
10 years grandson:
 Evidence of varicella immunity: nothing
 No Evidence of immunity:
varicella vaccine: within 3 days, and possibly up to 5 days, of
exposure to rash.
zoster immune globulin (ZIG): within 72 hours of exposure , 96
hours after exposure in Immuno compromised patients
CASE 10
6 months baby girl was brought to the out patient clinic
suffering of fever, running nose and fast breathing .Her mother
has fever, body aches, cough and sore throat for 2 days.
a) What is the differential diagnosis?
b) How to confirm diagnosis?
c) What are the measures taken for the girl and her contacts?
CASE 10
a) Differential Diagnosis:
CASE 10
Other less possible diagnosis:
1. otitis media: fever, ear pain, irritability, crying.
2. sinusitis: postnasal discharge, high fever, cough, headache.
3. Strept. Pharyngitis: high fever- sore throat-red oedmatous pharynx, tonsil
4. Catarrhal stage of measles, rubella
5. Pertussis: upper respiratory catarrh for 10 days, paroxysmal attack of spasmodic
cough (whooping cough)
b) Confirm diagnosis:
Clinically, the girl has influenza infection and early sign of pneumonia
The diagnosis base on symptoms, history of contact with flu patient and fast
breathing as early sign of pneumonia.
o A case definition of fever 100°F or greater and cough and/or sore throat is
used by CDC in Influenza-like Illness Surveillance Network (ILINet).
o Individual cases of influenza typically are not investigated. Exceptions to
this are severe, unusual complications. Or
o suspected or confirmed to be of animal origin (most frequently swine or
avian).
o Definitive diagnosis of influenza requires laboratory confirmation in
addition to signs and symptoms.
CASE 10
Measures to girl & contacts:
Girl: notification, isolation, disinfection, ttt
Treatment: Oseltamivir 3mg/kg twice daily for 5 days.
Contacts:
Chemo prophylaxis:
 Antiviral chemoprophylaxis generally is not recommended if more than 48
hours have elapsed since the first exposure to a person with influenza.
 once daily for 7 -14 days
 Oseltamivir:75 mg, oral, used for 3months and older.
 Zanamivir: 10 mg, inhaled , recommended from age 7 years and older.
Indication:
 people at high risk for complications who cannot receive influenza vaccine.
 people at high risk of influenza complications during the first two weeks following
vaccination.
 people with severe immune deficiencies or others who might not respond to influenza
vaccination.
2- vaccine: no post exposure prevention but still recommended if still in flu season.
CASE 11
3 years old boy visited the outpatient clinic with his mother
complaining from generalized rash first appear on face and
preceded by mild fever, on examination there was enlarged
cervical lymph node.
a) What is the possible diagnosis?
b) How to confirm?
c) What are measures taken for the child and house hold
contacts including pregnant sister?
CASE 11
a) diagnosis: probable rubella infection
b) Confirm diagnosis:
 Isolation of rubella virus; or
 Detection of rubella-virus specific nucleic acid by polymerase chain
reaction; or
 IgG seroconversion or a significant rise between acute- and
convalescent-phase titers in serum rubella IgG antibody level by
any standard serologic assay; or
 Positive serologic test for rubella IgM antibody†*
OR
 Epidemiologic linkage to a laboratory-confirmed case of rubella.
CASE 11
Case:
Isolation: for 5--7 days after rash onset.
Treatment : There is no specific antiviral therapy. Symptomatic, address
complications such as bacterial infections.
Contact:
Rubella vaccine and IG are not effective as PEP
MEASURES taken to prevent spread of infection:
 Identify and vaccinate susceptible persons who have no contraindications to
rubella vaccine.
 Ensure that pregnant women who are exposed to rubella are serologically
evaluated for rubella-specific IgM and IgG antibodies.
 Infection indicated by +ve IgM or rising titre of IgG or IgG seroconversion.
 Counsel susceptible pregnant women regarding the risks for intrauterine
rubella infection and recommend that they restrict their contact with persons
with confirmed, probable, or suspected rubella for >6 weeks (two incubation
periods) after rash onset in the last identified patient.
 Abortion is recommended, IG in high dose if the mother refuse abortion.
MCQ
There are reports of an outbreak of mumps in your
community. As you prepare to see patients who may
be infected, what should you consider regarding
mumps?
A. Two doses of the measles-mumps-rubella (MMR)
vaccine provides over 99% protection against mumps
B. Mumps is only communicable after the onset of parotitis
C. Up to one-quarter of individuals infected with mumps
virus may be asymptomatic
D. Public health policies in schools effectively eliminate the
risk of the spread of mumps
Key: C
MCQ
2. As regard a third dose of MMR as post exposure
prophylaxis. What is the policy of the Advisory
Committee on Immunization Practices (ACIP)?
A. It should be administered to household contacts only
B. It should be administered to children between the ages
of 5 and 12 years only
C. It should be applied broadly in affected communities
D. It should not be used at all
KEY: A
MCQ
What was the main result of using 3rd dose MMR as post
exposure prophylaxis against mumps?
A. MMR was not associated with a numerical or statistical
benefit
B. MMR was associated with a lower rate of secondary cases
of mumps, but the result was not statistically significant
C. MMR significantly reduced the risk of secondary cases of
mumps
D. MMR caused more cases of mumps than it prevented
Key: B
MCQ
4. The commonest cause of bacterial meningitis
in newborns is:
A. Group B Streptococcus
B. Streptococcus pneumoniae
C. Listeria monocytogenes
D. Escherichia coli
KEY: A
MCQ
4. The commonest cause of bacterial meningitis
in children is:
A. Streptococcus pneumoniae
B. Neisseria meningitidis
C. Haemophilus influenzae type b (Hib)
D. group B Streptococcus
KEY: A
MCQ
The most incriminated organism in causing
bacterial meningitis in Teens and young adults is:
A. Neisseria meningitidis
B. Streptococcus pneumoniae
C. Listeria monocytogenes
D. Haemophilus influenzae type b (Hib)
KEY: A
MCQ
The main causative organism of viral meningitis
is:
A. Non polio enterovirus
B. Mumps virus
C. Measles virus
D. Influenza virus
KEY: A
MCQ
A presumptive case of pulmonary TB in which there is
only 2 of 3 smears are positive for AFB. What is the
most appropriate action?
A. Do a third confirmatory smear
B. Initiate treatment
C. Ask for x ray
D. Ask for TB culture
KEY: B
MCQ
50 years old man complain from cough for 1 month,
anorexia, night sweat and fatigue. On sputum analysis,
only one of three smears was positive for AFB, what is
the suitable action?
A. Repeat sputum analysis
B. Begin treatment immediately
C. Ask for x ray
D. Consider another diagnosis
Key: c
MCQ
Tuberculin test is considered positive in
previously immunized person with BCG if
induration is:
A. ≥ 5 mm
B. ≥ 10 mm
C. ≥ 15 mm
D. 10 – 15 mm
Key: c
MCQ
All the following statements are wrong about tuberculin
test results except:
A. Indurations of ≥ 15 mm is positive in HIV positive non
immunized patient
B. Indurations of ≥ 15 mm is positive in healthy non
immunized individual.
C. Indurations of ≥ 10 mm is positive in children before age
of 5 years
D. Indurations of ≥ 10mm is positive in children after age of 6
years
Key: D
MCQ
A 5 year old child comes to the immunization centre without
BCG scar on his arm; what would you prefer?
a) Give BCG vaccine
b) Perform mantoux if positive then give BCG
c) No need of BCG
d) Chemoprophylaxis
e) Perform mantoux if negative then given BCG
Key: e
A mother brought her six weeks old child to an EPI
centre for routine immunization. She was enquired
about history of Epilepsy in the family and febrile fits.
The doctor took this history to avoid complication
with:
a) Diphtheria toxoid
b) Tetanus toxoid
c) Hepatitis B vaccine
d) Pertussis vaccine
e) OPV
Key: True: d
A 6 weeks old boy came for DPT, polio & HBV
vaccination. He was given initial doses of all and was
called after 4 weeks to have the next doses. The
likely reason for calling him again was:
a) Loss of immune memory
b) Stimulation of macrophages
c) Summation of immune responses
d) Replication of lymphocytes
e) Immune tolerance
Key: True: c
A mother brought her four year old child to the doctor. She
gave the history that her child was in close contact with a
case of diphtheria in school. She was very anxious about
her child and gave history of booster dose of DT 2 years
ago. What would be line of management for such a child?
a) Booster dose of DT with penicillin
b) Active and passive immunization
c) Active and passive immunization with chemoprophylaxis
d) Only keep under surveillance for 1 week
e) Nothing more required
Key: a
In Sir-Syed Model School a student of class 3
developed measles. The child was isolated from rest
of the class. The school medical officer advised for
the contacts of this child:
a) Active immunization within 3 days
b) Passive immunization
c) Chemoprophylaxis
d) Isolation
e) Anti-viral therapy
Key: b
A woman brought her child with congenital anomalies
of heart and cataract. She gave history of mild fever
and rash in the first trimester of pregnancy, which
settled with mild antipyretics. The possible gestational
condition that resulted in these anomalies was:
a) Measles
b) Herpes
c) Rubella
d) Drugs taken in pregnancy
e) Streptococcal scarlet fever
Key: True: c
A 22 years old man presented with painful, vesicular
and postulate eruption on his back. He gave history of
chicken pox infection at ten years of age. The likely
diagnosis was:
a) Measles
b) Meningococcemia
c) Herpes simplex
d) Scarlet fever
e) Herpes zoster
Key: True: e
There is an epidemic of Meningococcal Meningitis
among jail prisoners. The best chemoprophylaxis for
the protection of contacts is by giving:
a) Rifampicin
b) Chloramphenicol
c) Chloroquine
d) Doxycycline
e) Penicillin
Key: True: a
A 5 years old boy developed fever with typical “dew
drop rash” over his body. It appeared first on the trunk
and spread to arms and legs later; there were no
signs of neck stiffness and rigidity. What could be
likely diagnosis?
a) Small pox
b) Measles
c) Tanapox
d) Meningococcal meningitis
e) Chicken pox
Key: True: e
A school child is diagnosed to have chicken pox. He
should be isolated from other school children till:
a) The scabs fall off
b) Two days after the scabs are formed
c) Three days after the fever develops
d) Five days after the development of pustules
e) Seven days after the development of pustules
Key: True: a
A mother brought her 4 years old child with complaint of
sore throat, difficulty in swallowing and low grade fever. On
examination mild erythema and whitish membrane was
found on the posterior pharynx. The doctor diagnosed him
as a case of Diphtheric Mother gave history of complete
course of immunization. In addition to penicillin what would
be your line of management?
a) Supportive treatment
b) Passive immunization
c) Active immunization
d) Active plus passive immunization
e) Active and passive immunization plus Tracheostomy
Key: True: b
In Pediatric OPD the physician examined a 3 years
old child with low grade fever, mild erythema in the
throat and whitish membrane on the left side tonsil.
The cervical lymph node was palpable. The doctor
advised the mother to isolate the child for 7 days from
other contacts of less than 5 years old. The most
probable diagnosis is
a) Pharyngitis
b) Tonsillitis
c) Diphtheria
d) Acute Laryngitis
e) Whooping cough
Key: True: c
A student of nursery class developed mild fever along
with irritating cough gradually becoming paroxysmal
along with characteristic whoop. What you suggest
for how long the student should be isolated from the
class?
a) I week
b) 2 weeks
c) 3 weeks
d) 4 weeks
e) 6 weeks
Key: True: d
A 30 years old man presented in emergency in POF
hospital with complaint of severe headache, fever and
vomiting. On examination neck stiffness was found.
He has just returned from hajj and gives no history of
preceding ailment or injury. The most probable
diagnosis is
a) Meningitis
b) Tetanus
c) Brain abscess
d) Cerebral Malaria
e) Ischemic stroke
Key: True: a
If you being a field doctor in department of public
health, are given a task to visit a low socioeconomic
community of a slum and to give report about the
immunization status of the community against
Tuberculosis. The single most important clue to this
immunization is
a) Monteux test
b) Tuberculosis patients
c) BCG scar
d) X-ray chest
e) Sputum for AFB
Key: True: c
An epidemiologist was assigned to find out all the
cases, both new and old of T.B, in a slum located
near Islamabad during year 2007.Prevelance of
tuberculosis is confirmed by:
a) Mass miniature radiography
b) Sputum examination
c) Sputum culture
d) Tuberculin test
e) BCG scar mark
Key: True: c
A 10 years old boy was brought to a doctor in a
hospital with history of moderate fever with shivering
and abundant rash on the trunk and buccal mucosa.
On examination there were vesicles filled with clear
fluid on the trunk and legs. The physician told the
mother that child is suffering from
a) Herpes
b) Chicken pox
c) Rubella
d) German measles
e) Tanapox
Key: True: b
A pregnant lady reported to ante natal clinic with
signs and symptoms of Rubella during 28th week of
gestation. She was insisting for the induction of labor
because of fear of congenital malformations of fetus.
She was told by the doctor that Rubella does not
cause major abnormalities of fetus after
a) 8th week of pregnancy
b) 12th week of pregnancy
c) 16th week of pregnancy
d) 20th week of pregnancy
e) 24th week of pregnancy
Key: True: c
A 7 years old child presented with sore throat low
grade fever rash on face and posterior auricular and
cervical lymphadenopathy. The most probable
diagnosis is
a) Chicken pox
b) Small pox
c) Measles
d) Rubella
e) Mumps
Key: True: d
A 3rd year MBBS student of FJMC presented in ENT
OPD with complaint of sore throat, pain of swallowing
& low grade fever examination revealed erythema on
the pharynx and a whitish membrane on the pharynx
extending to the left tonsil. The doctor diagnosed her
as a case of diphtheria. What do you suggest
minimum isolation period:
a) Six daily negative throat and nasal swabs report
b) Till the signs & symptoms settle down
c) One week course of antibiotic
d) For days
e) Till complete blood picture becomes normal
Key: True: a
A mother brought her child with history of paroxysmal
cough and restlessness. On examination he showed
a loud inspiratory sound and sub-conjuctival
haemorrhagic On the basis of clinical presentation
what should be the drug of choice:
a) Erythromycin
b) Ampicillin
c) Tetracycline
d) Sulphadiazine
e) Co- trimoxazole
Key: a
A 40 year old tuberculosis patient on ATT for the last
two months presented to his physician with
complaints of tingling, numbness and loss of
peripheral sensation. The likely anti tuberculosis drug
to have caused these symptoms is:
a) Isoniazid
b) Rifampicin
c) Streptomycin
d) Pyrazinamide
e) Ethambutol
Key: a
10 years old boy presented with high grade fever,
chills, aches, cough and generalized weakness. He
was diagnosed as a case of influenza. The most
dreaded complication is:
a) Encephalitis
b) Pneumonia
c) Toxic shock syndrome
d) Reye’s syndrome
e) Sub-Conjunctival hemorrhages
Key: b
A 22 years old married non pregnant woman
developed rubella infection. In order to avoid
congenital rubella syndrome in her pregnancy she
should be given:
a) Antibiotics
b) Active immunization
c) Nonspecific immunization
d) Advice to avoid conception for 12 weeks
e) Anti viral therapy
Key: d
A 40 years old man was diagnosed as a case of TB 4
weeks ago. He has been taking ATT for the last 3
weeks. His sputum analysis showed AFB on follow up
investigation. Such a case of TB is known as:
a) Failure case
b) Newer case
c) Defaulter
d) Transfer out
e) Resistant
Key: b
A primigravida presented in Medical OPD at 39
weeks of gestation with dew drop rash on the body
for 1 day she was diagnosed as having chicken pox
she was told that her baby is at higher risk of having:
a) Low birth weight
b) Microcephaly
c) Atrophied limbs
d) Varicella infections
e) Cutaneous scars
Key: d
A 17 year old boy was brought in emergency
department with symptoms of acute encephalopathy.
He was admitted in ICU, initial investigation revealed
that his liver had undergone fatly degeneration. His
father gave history of rash on his body. Most likely he
suffered from:
a) Measles
b) Rubella
c) Chicken pox
d) Mumps
e) Cutaneous diphtheria
Key: c
After serial sonography it was told to the apparently
healthy pregnant woman that her baby is
microcephalic and of low birth weight she gave no
important medical history of note except mild febrile
illness with rapidly disappearing rash in 2nd month of
pregnancy. Most likely she suffered from:
a) Chicken pox
b) German measles
c) Measles
d) Cutaneous diphtheria
e) Malaria
Key: b
4 years old girl was having fever, cough with a
characteristic whoop. She was diagnosed as a case
of whooping cough several antibodies are effective
they are important as they:
a) Reduce the frequency of spasm
b) Control severity of disease
c) Shorten the illness
d) Control secondary bacterial infection
e) Prevent carrier state
Key: d
A child of 6 presented to school medical officer with
complains of fever, malaise and painful swallowing.
On examination a diffuse swelling was observed on
the sides of the face below and in-front of the ears.
The doctor diagnosed him as a case of mumps. What
is the most appropriate management for him?
a) Analgesics only
b) Active and passive immunization
c) Passive immunization
d) Antibiotics only
e) Rest, analgesics and balanced diet
Key: e
50 years old Bank officer was told to be suffering from
illeocecal Tuberculosis. Nobody suffered tuberculosis
in the family, but he still got infected, probably due to:
A. Infected milk
B. Infected clients
C. Contaminated vessel
D. Stressful work
E. Advancing age
Key: a
44 years old man presented with fever, cough, night
sweats and weight loss. He is HIV positive , has crackles
on both lungs and chest x ray shows bilateral lower lobe
infiltrate. All the following are correct except:
A. Evaluation should include sputum gram stain and culture
B. Sputum should be stained and cultured for M.
tuberculosis
C. Patient require pneumocystis pneumonia prophylaxis
D. Infiltrates in lower lobes exclude diagnosis of TB
KEY: D
Which of the following groups should not receive live
attenuated influenza vaccine:
A. Adults with mild to moderate illness
B. Healthy children 2- 5 years
C. Healthy pregnant
D. Penicillin allergic adults
Key: c
Which statements about Tdap vaccine is false:
A. Tdap is contraindicated in pregnancy
B. Tdap should replace a single dose of td vaccine
for adults who don’t receive a dose of Tdap and
require a booster of TD
C. Tdap is not licensed for use among adults aged
65 and more
D. Tdap is contraindicated in adults allergic to
formaldehyde
Key: A
A 20 month old child sent to your office with mild viral
infection . Result of examination was normal except for
temperature 37.2 c and clear nasal discharge. She
received 2 doses of DTap, OPV and she did not take MMR
vaccine. The mother is 20 week pregnant and her brother
undergoing chemotherapy for leukemia. Which is more
appropriate intervention.
A. Scheduale a visit in two weeks for DTaP
B. Administer inactivated polio (IPV) and DTaP
C. Administer DTaP, opv amd MMR
D. Administer DTaP, IPV amd MMR
E. Administer DTaP, opv amd MMR after 3 months
Key: D
Immunization of preschool children with diphtheria toxoid
result in:
A. Protection against the diphtheria carrier state
B. Lifelong immunity against diphtheria
C. Detectable antitoxin or immunologic memory for 10 years
D. Frequent adverse effects
E. Protection against infection with C. diphtheria
Key: c
What is the recommended interval in months between the
administration of whole blood transfusion and MMR
vaccine?
A. 0
B. 1
C. 3
D. 6
E. 10
Key: D
The most common opportunistic infections
occuring in HIV patients is:
A. Tuberculosis
B. Pneumocystis pneumonia
C. Amoebic encephalitis
D. Fungal pneumonia
Key: A
A 2 years old boy is brought to ER with sever prostration,
temperature 40 c and few petechial lesions around the
ankle. A gram stian of blood showed gram negative
diplococci , what is the case fatality rate of this disease?
A. 5 – 15 %
B. 20 – 30 %
C. 40 – 50 %
D. More than 50%
Key: A
CONTINUE
Compare to general population the risk of
developing infection in household contacts is:
A. the same
B. 10 – 20 time more
C. 50 -100 time more
D. 200 -400 time more
E. 500 – 800 time more
key: E
CONTINUE
The child had been to child day care center, in addition to
close surveillance, which of the following is most
appropriate intervention?
A. No further action
B. Vaccination of children only
C. Vaccination of children and adults
D. Antibiotic prophylaxis for children only
E. Antibiotic prophylaxis for children and adults
Key: E
A 7 years old girl brought to your office because of a
rash that appeared 3 days ago. Her temp. was 37.2 c
and her face has intense rash with a slapped cheek
appearance. The most likely etiological factor is:
A. Adenovirus
B. Rotavirus
C. Parvovirus
D. Coxsackie virus
E. Echovirus
Key: c
To which patient MMR would be safe to administer?
A. A 15 months old HIV infected child with CD4 cell count
700
B. A 25 years old pregnant woman
C. A 12 years old asthmatic on 20 mg of oral prednisone
daily for the last 20 days
D. A 18 years old leukemia in remission whose
chemotherapy was terminated 1 month ago
E. A 17 years old with life threatening anaphylactic reaction
to egg
Key: A
A 19 years old colleague student complaining of sever
coughing spells for the last 4 days following initial
symptoms of coryza and malaise. She is afebrile. In
weekends she baby sits a 10 month and 2 years old
children, in term of controlling contacts , What is the most
important etiological factor to be included in D.D?
A. Streptococcus pneumoniae
B. Mycoplasma pneumoniae
C. Bordetella pertussis
D. Influenza virus
E. Legionella
Key: c
Which of the following infections is transmitted mainly
from person to person?
A. California encephalitis
B. St. louis encephalitis
C. West Nile viral encephalitis
D. Meningococcal meningitis
E. Eastern equine encephalitis
Key: D
In investigation about influenza outbreak, you found out
that a large number of persons who developed mild
symptoms, have been vaccinated with trivalent vaccine
containing the appropriate strain for the current year. The
isolated strain is the same strain in the trivalent vaccine.
What is the most likely explanation?
A. Vaccine failure
B. Antigenic drift
C. Antigenic shift
D. Herd immunity
E. Incomplete immunity due to rhinovirus infection
Key: B
A 5 years old preschool child presented with fever,
malaise and vesicular rash that started 24 h ago. He has a
sibling 3 months old and pregnant mother 38 week both
are susceptible. What is the most common complication?
A. Pneumonia
B. Reyes syndrome
C. Encephalitis
D. Orchitis
E. Thrombocytopenia
Key: A
CONTINUE
What is the most appropriate management for contacts?
A. Observation only for all contacts
B. Vaccination for the mother, sibling and class mates
C. Immune globulins for mother, sibling and class mates
D. Immune globulins for mother and vaccination for sibling
and class mates
E. Immune globulins for mother and sibling and vaccination
for class mates
Key: E
Under which condition should chemoprophylaxis for influenza
be considered?
A. All nursing home residents and unvaccinated staff during an
influenza A outbreak
B. All nursing home residents and unvaccinated staff during an
influenza B outbreak
C. Only unvaccinated nursing home residents and staff during an
influenza A outbreak
D. Only unvaccinated nursing home residents and staff during an
influenza B outbreak
E. All nursing home residents and staff during an influenza B
outbreak
Key: A
For which patient pneumococcal vaccine PPV23 is not
beneficial?
A. A 15 month old HIV infected child
B. A 20 years old about to undergo splenectomy for ITP
C. A 70 years old healthy female
D. A 5 years old child with sickle cell disease
E. A 10 years old boy with nephrotic syndrome who
received the vaccine 5 years ago.
Key: A
A 32 years old HIV infected patient, has 5 mm induration
after tuberculin test. His chest x ray is normal, he is
currently on protease inhibitors antiretroviral treatment.
He did not receive anti tuberculosis treatment nor he had
been in contact with tuberculosis patient. What is the
most appropriate intervention?
A. No preventive therapy for TB
B. Isoniazid for 9 months
C. Rifampicin for 9 months
D. Rifampicin and pyrazinamide for 2 months
E. Streptomycin for 6 months
Key: B
HIV infected patients have high risk of developing active
TB resistant to:
A. Isoniazid
B. Rifampicin
C. Streptomycin
D. Pyrazinamide
E. Ethambutole
Key: A
Four drug therapy is recommended as initial approach
to HIV infected active TB patient:
A. Always
B. When multidrug resistant TB exceed 4% in community
C. When the patient had anti tuberculosis treatment before
D. When the patient has contact with multidrug resistant TB
E. When CD4 count is under 200
Key: A
A 30 years old farmer suffering from fever, coughing,
night sweats and malaise. He received BCG when he
was child and his skin test is 15 mm, the most likely
diagnosis;
A. Influenza
B. Brucellosis
C. Aspergillosis
D. Mycobacterium tuberculosis
E. Mycobacterium bovis
Key: E
A 4 years old girl suffering from sore throat, fever,
hoarseness and drolling. What is the most likely
diagnosis?
A. Poliovirus
B. C. Tetani
C. C. Diphtheria
D. H. Influenza
E. Streptococci
Key: D

Más contenido relacionado

La actualidad más candente (20)

Yersinia pestis presentation
Yersinia pestis presentationYersinia pestis presentation
Yersinia pestis presentation
 
Difference between a pandemic, an epidemic, endemic, and an outbreak
Difference between a pandemic, an epidemic, endemic, and an outbreakDifference between a pandemic, an epidemic, endemic, and an outbreak
Difference between a pandemic, an epidemic, endemic, and an outbreak
 
H1 N1 influenza (swine flu)
H1 N1 influenza (swine flu)H1 N1 influenza (swine flu)
H1 N1 influenza (swine flu)
 
Small pox
Small poxSmall pox
Small pox
 
Tuberculosis
TuberculosisTuberculosis
Tuberculosis
 
Water borne diseases
Water borne diseasesWater borne diseases
Water borne diseases
 
Rickettsia
RickettsiaRickettsia
Rickettsia
 
Vector borne disease
Vector borne diseaseVector borne disease
Vector borne disease
 
Mycobacterium tuberculosis(Microbiology)
Mycobacterium tuberculosis(Microbiology)Mycobacterium tuberculosis(Microbiology)
Mycobacterium tuberculosis(Microbiology)
 
Water-Borne Diseases and its Prevention
Water-Borne Diseases and its PreventionWater-Borne Diseases and its Prevention
Water-Borne Diseases and its Prevention
 
Modes of transmission
Modes of transmissionModes of transmission
Modes of transmission
 
Zoonotic infection
Zoonotic infectionZoonotic infection
Zoonotic infection
 
Yersinia pestis .....
Yersinia pestis .....Yersinia pestis .....
Yersinia pestis .....
 
Prevention and control of infectious diseases
Prevention and control of infectious diseasesPrevention and control of infectious diseases
Prevention and control of infectious diseases
 
Dengue virus
Dengue virus  Dengue virus
Dengue virus
 
Influenza
Influenza Influenza
Influenza
 
Infleunza
InfleunzaInfleunza
Infleunza
 
Water borne diseases
Water borne diseasesWater borne diseases
Water borne diseases
 
H1 N1
H1 N1H1 N1
H1 N1
 
Brucella
BrucellaBrucella
Brucella
 

Similar a Air borne diseases

Lecture 10. diphtheria
Lecture 10. diphtheriaLecture 10. diphtheria
Lecture 10. diphtheriaVasyl Sorokhan
 
Communicable diseases
Communicable diseasesCommunicable diseases
Communicable diseasesAnju T.N
 
acute respiratory infection ppt-140416232653-phpapp01.pptx
acute respiratory infection ppt-140416232653-phpapp01.pptxacute respiratory infection ppt-140416232653-phpapp01.pptx
acute respiratory infection ppt-140416232653-phpapp01.pptxcitymdc
 
ACUTE AND CHRONIC CONDITION OF PHARYNX & LARYNX.ppt
ACUTE AND CHRONIC CONDITION OF PHARYNX & LARYNX.pptACUTE AND CHRONIC CONDITION OF PHARYNX & LARYNX.ppt
ACUTE AND CHRONIC CONDITION OF PHARYNX & LARYNX.pptDrBPSah
 
Paediatric respiratory problems
Paediatric respiratory problemsPaediatric respiratory problems
Paediatric respiratory problemsmedicostest
 
acute respiratory tract infection
acute respiratory tract infectionacute respiratory tract infection
acute respiratory tract infectionAnwar Ahmad
 
PNEUMONIA PATHOPHYSIOLOGY.pptx
PNEUMONIA PATHOPHYSIOLOGY.pptxPNEUMONIA PATHOPHYSIOLOGY.pptx
PNEUMONIA PATHOPHYSIOLOGY.pptxArunKumarP478781
 
GR 6 MUMPS AND NCROUPS.pptx2222222222222
GR 6 MUMPS AND NCROUPS.pptx2222222222222GR 6 MUMPS AND NCROUPS.pptx2222222222222
GR 6 MUMPS AND NCROUPS.pptx2222222222222KelfalaHassanDawoh
 
4 child health care (2).pptx
4 child health care (2).pptx4 child health care (2).pptx
4 child health care (2).pptxTatenufAlemayehu
 
New PPT Presentation.pptx
New PPT Presentation.pptxNew PPT Presentation.pptx
New PPT Presentation.pptxssuseref3feb
 
Respiratory disease
Respiratory diseaseRespiratory disease
Respiratory diseaseManoj Mahato
 

Similar a Air borne diseases (20)

Lecture 10. diphtheria
Lecture 10. diphtheriaLecture 10. diphtheria
Lecture 10. diphtheria
 
COMMUNIABLE DISEASE.pptx
COMMUNIABLE DISEASE.pptxCOMMUNIABLE DISEASE.pptx
COMMUNIABLE DISEASE.pptx
 
Communicable diseases
Communicable diseasesCommunicable diseases
Communicable diseases
 
acute respiratory infection ppt-140416232653-phpapp01.pptx
acute respiratory infection ppt-140416232653-phpapp01.pptxacute respiratory infection ppt-140416232653-phpapp01.pptx
acute respiratory infection ppt-140416232653-phpapp01.pptx
 
ACUTE AND CHRONIC CONDITION OF PHARYNX & LARYNX.ppt
ACUTE AND CHRONIC CONDITION OF PHARYNX & LARYNX.pptACUTE AND CHRONIC CONDITION OF PHARYNX & LARYNX.ppt
ACUTE AND CHRONIC CONDITION OF PHARYNX & LARYNX.ppt
 
Paediatric respiratory problems
Paediatric respiratory problemsPaediatric respiratory problems
Paediatric respiratory problems
 
acute respiratory tract infection
acute respiratory tract infectionacute respiratory tract infection
acute respiratory tract infection
 
RTIs
RTIsRTIs
RTIs
 
PNEUMONIA PATHOPHYSIOLOGY.pptx
PNEUMONIA PATHOPHYSIOLOGY.pptxPNEUMONIA PATHOPHYSIOLOGY.pptx
PNEUMONIA PATHOPHYSIOLOGY.pptx
 
1. Acute Resp dzs
1. Acute Resp dzs1. Acute Resp dzs
1. Acute Resp dzs
 
4)chronic pharyngeal infections
4)chronic pharyngeal infections4)chronic pharyngeal infections
4)chronic pharyngeal infections
 
GR 6 MUMPS AND NCROUPS.pptx2222222222222
GR 6 MUMPS AND NCROUPS.pptx2222222222222GR 6 MUMPS AND NCROUPS.pptx2222222222222
GR 6 MUMPS AND NCROUPS.pptx2222222222222
 
4 child health care (2).pptx
4 child health care (2).pptx4 child health care (2).pptx
4 child health care (2).pptx
 
Pneumonia
PneumoniaPneumonia
Pneumonia
 
New PPT Presentation.pptx
New PPT Presentation.pptxNew PPT Presentation.pptx
New PPT Presentation.pptx
 
Acute respiratory infection (ARI)
Acute respiratory infection (ARI)Acute respiratory infection (ARI)
Acute respiratory infection (ARI)
 
URTI PC I.pptx
URTI PC I.pptxURTI PC I.pptx
URTI PC I.pptx
 
Pertussis
PertussisPertussis
Pertussis
 
Respiratory disease
Respiratory diseaseRespiratory disease
Respiratory disease
 
Diphtheria.pptx
Diphtheria.pptxDiphtheria.pptx
Diphtheria.pptx
 

Más de monaaboserea

Refugee &migrants health
Refugee &migrants healthRefugee &migrants health
Refugee &migrants healthmonaaboserea
 
Arthropod borne mc qs
Arthropod borne  mc qsArthropod borne  mc qs
Arthropod borne mc qsmonaaboserea
 
Cases of-arthropod-borne-diseases
Cases of-arthropod-borne-diseasesCases of-arthropod-borne-diseases
Cases of-arthropod-borne-diseasesmonaaboserea
 
Arthropod borne part 2
Arthropod borne part 2Arthropod borne part 2
Arthropod borne part 2monaaboserea
 
Arthropod. borne part 1
Arthropod. borne  part 1Arthropod. borne  part 1
Arthropod. borne part 1monaaboserea
 
Research methodology
Research methodologyResearch methodology
Research methodologymonaaboserea
 
Non communicable diseases part 1
Non communicable diseases part 1Non communicable diseases part 1
Non communicable diseases part 1monaaboserea
 
Mcqs general epidemiology
Mcqs   general epidemiologyMcqs   general epidemiology
Mcqs general epidemiologymonaaboserea
 
General epidemiology
General epidemiologyGeneral epidemiology
General epidemiologymonaaboserea
 
Disaster and-communicable-diseases
Disaster and-communicable-diseasesDisaster and-communicable-diseases
Disaster and-communicable-diseasesmonaaboserea
 

Más de monaaboserea (20)

Prisoners' health
Prisoners' healthPrisoners' health
Prisoners' health
 
Traveler medicine
Traveler medicineTraveler medicine
Traveler medicine
 
Refugee &migrants health
Refugee &migrants healthRefugee &migrants health
Refugee &migrants health
 
Arthropod borne mc qs
Arthropod borne  mc qsArthropod borne  mc qs
Arthropod borne mc qs
 
Cases of-arthropod-borne-diseases
Cases of-arthropod-borne-diseasesCases of-arthropod-borne-diseases
Cases of-arthropod-borne-diseases
 
Arthropod borne part 2
Arthropod borne part 2Arthropod borne part 2
Arthropod borne part 2
 
Arthropod. borne part 1
Arthropod. borne  part 1Arthropod. borne  part 1
Arthropod. borne part 1
 
Screening tests
Screening  testsScreening  tests
Screening tests
 
Sampling
SamplingSampling
Sampling
 
Risk assessment
Risk assessmentRisk assessment
Risk assessment
 
Research methodology
Research methodologyResearch methodology
Research methodology
 
Statistics
StatisticsStatistics
Statistics
 
Ncds part 2
Ncds part 2Ncds part 2
Ncds part 2
 
Non communicable diseases part 1
Non communicable diseases part 1Non communicable diseases part 1
Non communicable diseases part 1
 
Mcqs general epidemiology
Mcqs   general epidemiologyMcqs   general epidemiology
Mcqs general epidemiology
 
General epidemiology
General epidemiologyGeneral epidemiology
General epidemiology
 
Disaster and-communicable-diseases
Disaster and-communicable-diseasesDisaster and-communicable-diseases
Disaster and-communicable-diseases
 
Substance abuse
Substance abuseSubstance abuse
Substance abuse
 
Mcqs food borne
Mcqs food borneMcqs food borne
Mcqs food borne
 
Aids issues
Aids  issuesAids  issues
Aids issues
 

Último

Lucknow Call girls - 8800925952 - 24x7 service with hotel room
Lucknow Call girls - 8800925952 - 24x7 service with hotel roomLucknow Call girls - 8800925952 - 24x7 service with hotel room
Lucknow Call girls - 8800925952 - 24x7 service with hotel roomdiscovermytutordmt
 
Vip Call Girls Anna Salai Chennai 👉 8250192130 ❣️💯 Top Class Girls Available
Vip Call Girls Anna Salai Chennai 👉 8250192130 ❣️💯 Top Class Girls AvailableVip Call Girls Anna Salai Chennai 👉 8250192130 ❣️💯 Top Class Girls Available
Vip Call Girls Anna Salai Chennai 👉 8250192130 ❣️💯 Top Class Girls AvailableNehru place Escorts
 
Low Rate Call Girls Kochi Anika 8250192130 Independent Escort Service Kochi
Low Rate Call Girls Kochi Anika 8250192130 Independent Escort Service KochiLow Rate Call Girls Kochi Anika 8250192130 Independent Escort Service Kochi
Low Rate Call Girls Kochi Anika 8250192130 Independent Escort Service KochiSuhani Kapoor
 
VIP Service Call Girls Sindhi Colony 📳 7877925207 For 18+ VIP Call Girl At Th...
VIP Service Call Girls Sindhi Colony 📳 7877925207 For 18+ VIP Call Girl At Th...VIP Service Call Girls Sindhi Colony 📳 7877925207 For 18+ VIP Call Girl At Th...
VIP Service Call Girls Sindhi Colony 📳 7877925207 For 18+ VIP Call Girl At Th...jageshsingh5554
 
Call Girls Ludhiana Just Call 9907093804 Top Class Call Girl Service Available
Call Girls Ludhiana Just Call 9907093804 Top Class Call Girl Service AvailableCall Girls Ludhiana Just Call 9907093804 Top Class Call Girl Service Available
Call Girls Ludhiana Just Call 9907093804 Top Class Call Girl Service AvailableDipal Arora
 
Call Girls Service Surat Samaira ❤️🍑 8250192130 👄 Independent Escort Service ...
Call Girls Service Surat Samaira ❤️🍑 8250192130 👄 Independent Escort Service ...Call Girls Service Surat Samaira ❤️🍑 8250192130 👄 Independent Escort Service ...
Call Girls Service Surat Samaira ❤️🍑 8250192130 👄 Independent Escort Service ...CALL GIRLS
 
Call Girls Varanasi Just Call 9907093804 Top Class Call Girl Service Available
Call Girls Varanasi Just Call 9907093804 Top Class Call Girl Service AvailableCall Girls Varanasi Just Call 9907093804 Top Class Call Girl Service Available
Call Girls Varanasi Just Call 9907093804 Top Class Call Girl Service AvailableDipal Arora
 
Premium Call Girls Cottonpet Whatsapp 7001035870 Independent Escort Service
Premium Call Girls Cottonpet Whatsapp 7001035870 Independent Escort ServicePremium Call Girls Cottonpet Whatsapp 7001035870 Independent Escort Service
Premium Call Girls Cottonpet Whatsapp 7001035870 Independent Escort Servicevidya singh
 
Bangalore Call Girl Whatsapp Number 100% Complete Your Sexual Needs
Bangalore Call Girl Whatsapp Number 100% Complete Your Sexual NeedsBangalore Call Girl Whatsapp Number 100% Complete Your Sexual Needs
Bangalore Call Girl Whatsapp Number 100% Complete Your Sexual NeedsGfnyt
 
Call Girls Aurangabad Just Call 9907093804 Top Class Call Girl Service Available
Call Girls Aurangabad Just Call 9907093804 Top Class Call Girl Service AvailableCall Girls Aurangabad Just Call 9907093804 Top Class Call Girl Service Available
Call Girls Aurangabad Just Call 9907093804 Top Class Call Girl Service AvailableDipal Arora
 
Call Girls Dehradun Just Call 9907093804 Top Class Call Girl Service Available
Call Girls Dehradun Just Call 9907093804 Top Class Call Girl Service AvailableCall Girls Dehradun Just Call 9907093804 Top Class Call Girl Service Available
Call Girls Dehradun Just Call 9907093804 Top Class Call Girl Service AvailableDipal Arora
 
(👑VVIP ISHAAN ) Russian Call Girls Service Navi Mumbai🖕9920874524🖕Independent...
(👑VVIP ISHAAN ) Russian Call Girls Service Navi Mumbai🖕9920874524🖕Independent...(👑VVIP ISHAAN ) Russian Call Girls Service Navi Mumbai🖕9920874524🖕Independent...
(👑VVIP ISHAAN ) Russian Call Girls Service Navi Mumbai🖕9920874524🖕Independent...Taniya Sharma
 
Best Rate (Hyderabad) Call Girls Jahanuma ⟟ 8250192130 ⟟ High Class Call Girl...
Best Rate (Hyderabad) Call Girls Jahanuma ⟟ 8250192130 ⟟ High Class Call Girl...Best Rate (Hyderabad) Call Girls Jahanuma ⟟ 8250192130 ⟟ High Class Call Girl...
Best Rate (Hyderabad) Call Girls Jahanuma ⟟ 8250192130 ⟟ High Class Call Girl...astropune
 
High Profile Call Girls Coimbatore Saanvi☎️ 8250192130 Independent Escort Se...
High Profile Call Girls Coimbatore Saanvi☎️  8250192130 Independent Escort Se...High Profile Call Girls Coimbatore Saanvi☎️  8250192130 Independent Escort Se...
High Profile Call Girls Coimbatore Saanvi☎️ 8250192130 Independent Escort Se...narwatsonia7
 
Call Girls Coimbatore Just Call 9907093804 Top Class Call Girl Service Available
Call Girls Coimbatore Just Call 9907093804 Top Class Call Girl Service AvailableCall Girls Coimbatore Just Call 9907093804 Top Class Call Girl Service Available
Call Girls Coimbatore Just Call 9907093804 Top Class Call Girl Service AvailableDipal Arora
 
Call Girls Jabalpur Just Call 9907093804 Top Class Call Girl Service Available
Call Girls Jabalpur Just Call 9907093804 Top Class Call Girl Service AvailableCall Girls Jabalpur Just Call 9907093804 Top Class Call Girl Service Available
Call Girls Jabalpur Just Call 9907093804 Top Class Call Girl Service AvailableDipal Arora
 
Russian Escorts Girls Nehru Place ZINATHI 🔝9711199012 ☪ 24/7 Call Girls Delhi
Russian Escorts Girls  Nehru Place ZINATHI 🔝9711199012 ☪ 24/7 Call Girls DelhiRussian Escorts Girls  Nehru Place ZINATHI 🔝9711199012 ☪ 24/7 Call Girls Delhi
Russian Escorts Girls Nehru Place ZINATHI 🔝9711199012 ☪ 24/7 Call Girls DelhiAlinaDevecerski
 
Top Rated Bangalore Call Girls Mg Road ⟟ 8250192130 ⟟ Call Me For Genuine Sex...
Top Rated Bangalore Call Girls Mg Road ⟟ 8250192130 ⟟ Call Me For Genuine Sex...Top Rated Bangalore Call Girls Mg Road ⟟ 8250192130 ⟟ Call Me For Genuine Sex...
Top Rated Bangalore Call Girls Mg Road ⟟ 8250192130 ⟟ Call Me For Genuine Sex...narwatsonia7
 
(Rocky) Jaipur Call Girl - 09521753030 Escorts Service 50% Off with Cash ON D...
(Rocky) Jaipur Call Girl - 09521753030 Escorts Service 50% Off with Cash ON D...(Rocky) Jaipur Call Girl - 09521753030 Escorts Service 50% Off with Cash ON D...
(Rocky) Jaipur Call Girl - 09521753030 Escorts Service 50% Off with Cash ON D...indiancallgirl4rent
 

Último (20)

Lucknow Call girls - 8800925952 - 24x7 service with hotel room
Lucknow Call girls - 8800925952 - 24x7 service with hotel roomLucknow Call girls - 8800925952 - 24x7 service with hotel room
Lucknow Call girls - 8800925952 - 24x7 service with hotel room
 
Vip Call Girls Anna Salai Chennai 👉 8250192130 ❣️💯 Top Class Girls Available
Vip Call Girls Anna Salai Chennai 👉 8250192130 ❣️💯 Top Class Girls AvailableVip Call Girls Anna Salai Chennai 👉 8250192130 ❣️💯 Top Class Girls Available
Vip Call Girls Anna Salai Chennai 👉 8250192130 ❣️💯 Top Class Girls Available
 
Low Rate Call Girls Kochi Anika 8250192130 Independent Escort Service Kochi
Low Rate Call Girls Kochi Anika 8250192130 Independent Escort Service KochiLow Rate Call Girls Kochi Anika 8250192130 Independent Escort Service Kochi
Low Rate Call Girls Kochi Anika 8250192130 Independent Escort Service Kochi
 
VIP Service Call Girls Sindhi Colony 📳 7877925207 For 18+ VIP Call Girl At Th...
VIP Service Call Girls Sindhi Colony 📳 7877925207 For 18+ VIP Call Girl At Th...VIP Service Call Girls Sindhi Colony 📳 7877925207 For 18+ VIP Call Girl At Th...
VIP Service Call Girls Sindhi Colony 📳 7877925207 For 18+ VIP Call Girl At Th...
 
Call Girls Ludhiana Just Call 9907093804 Top Class Call Girl Service Available
Call Girls Ludhiana Just Call 9907093804 Top Class Call Girl Service AvailableCall Girls Ludhiana Just Call 9907093804 Top Class Call Girl Service Available
Call Girls Ludhiana Just Call 9907093804 Top Class Call Girl Service Available
 
Call Girls Service Surat Samaira ❤️🍑 8250192130 👄 Independent Escort Service ...
Call Girls Service Surat Samaira ❤️🍑 8250192130 👄 Independent Escort Service ...Call Girls Service Surat Samaira ❤️🍑 8250192130 👄 Independent Escort Service ...
Call Girls Service Surat Samaira ❤️🍑 8250192130 👄 Independent Escort Service ...
 
Call Girls Varanasi Just Call 9907093804 Top Class Call Girl Service Available
Call Girls Varanasi Just Call 9907093804 Top Class Call Girl Service AvailableCall Girls Varanasi Just Call 9907093804 Top Class Call Girl Service Available
Call Girls Varanasi Just Call 9907093804 Top Class Call Girl Service Available
 
Premium Call Girls Cottonpet Whatsapp 7001035870 Independent Escort Service
Premium Call Girls Cottonpet Whatsapp 7001035870 Independent Escort ServicePremium Call Girls Cottonpet Whatsapp 7001035870 Independent Escort Service
Premium Call Girls Cottonpet Whatsapp 7001035870 Independent Escort Service
 
Bangalore Call Girl Whatsapp Number 100% Complete Your Sexual Needs
Bangalore Call Girl Whatsapp Number 100% Complete Your Sexual NeedsBangalore Call Girl Whatsapp Number 100% Complete Your Sexual Needs
Bangalore Call Girl Whatsapp Number 100% Complete Your Sexual Needs
 
Call Girls Aurangabad Just Call 9907093804 Top Class Call Girl Service Available
Call Girls Aurangabad Just Call 9907093804 Top Class Call Girl Service AvailableCall Girls Aurangabad Just Call 9907093804 Top Class Call Girl Service Available
Call Girls Aurangabad Just Call 9907093804 Top Class Call Girl Service Available
 
Call Girls Dehradun Just Call 9907093804 Top Class Call Girl Service Available
Call Girls Dehradun Just Call 9907093804 Top Class Call Girl Service AvailableCall Girls Dehradun Just Call 9907093804 Top Class Call Girl Service Available
Call Girls Dehradun Just Call 9907093804 Top Class Call Girl Service Available
 
(👑VVIP ISHAAN ) Russian Call Girls Service Navi Mumbai🖕9920874524🖕Independent...
(👑VVIP ISHAAN ) Russian Call Girls Service Navi Mumbai🖕9920874524🖕Independent...(👑VVIP ISHAAN ) Russian Call Girls Service Navi Mumbai🖕9920874524🖕Independent...
(👑VVIP ISHAAN ) Russian Call Girls Service Navi Mumbai🖕9920874524🖕Independent...
 
Best Rate (Hyderabad) Call Girls Jahanuma ⟟ 8250192130 ⟟ High Class Call Girl...
Best Rate (Hyderabad) Call Girls Jahanuma ⟟ 8250192130 ⟟ High Class Call Girl...Best Rate (Hyderabad) Call Girls Jahanuma ⟟ 8250192130 ⟟ High Class Call Girl...
Best Rate (Hyderabad) Call Girls Jahanuma ⟟ 8250192130 ⟟ High Class Call Girl...
 
High Profile Call Girls Coimbatore Saanvi☎️ 8250192130 Independent Escort Se...
High Profile Call Girls Coimbatore Saanvi☎️  8250192130 Independent Escort Se...High Profile Call Girls Coimbatore Saanvi☎️  8250192130 Independent Escort Se...
High Profile Call Girls Coimbatore Saanvi☎️ 8250192130 Independent Escort Se...
 
Call Girls Coimbatore Just Call 9907093804 Top Class Call Girl Service Available
Call Girls Coimbatore Just Call 9907093804 Top Class Call Girl Service AvailableCall Girls Coimbatore Just Call 9907093804 Top Class Call Girl Service Available
Call Girls Coimbatore Just Call 9907093804 Top Class Call Girl Service Available
 
Call Girls Jabalpur Just Call 9907093804 Top Class Call Girl Service Available
Call Girls Jabalpur Just Call 9907093804 Top Class Call Girl Service AvailableCall Girls Jabalpur Just Call 9907093804 Top Class Call Girl Service Available
Call Girls Jabalpur Just Call 9907093804 Top Class Call Girl Service Available
 
Russian Call Girls in Delhi Tanvi ➡️ 9711199012 💋📞 Independent Escort Service...
Russian Call Girls in Delhi Tanvi ➡️ 9711199012 💋📞 Independent Escort Service...Russian Call Girls in Delhi Tanvi ➡️ 9711199012 💋📞 Independent Escort Service...
Russian Call Girls in Delhi Tanvi ➡️ 9711199012 💋📞 Independent Escort Service...
 
Russian Escorts Girls Nehru Place ZINATHI 🔝9711199012 ☪ 24/7 Call Girls Delhi
Russian Escorts Girls  Nehru Place ZINATHI 🔝9711199012 ☪ 24/7 Call Girls DelhiRussian Escorts Girls  Nehru Place ZINATHI 🔝9711199012 ☪ 24/7 Call Girls Delhi
Russian Escorts Girls Nehru Place ZINATHI 🔝9711199012 ☪ 24/7 Call Girls Delhi
 
Top Rated Bangalore Call Girls Mg Road ⟟ 8250192130 ⟟ Call Me For Genuine Sex...
Top Rated Bangalore Call Girls Mg Road ⟟ 8250192130 ⟟ Call Me For Genuine Sex...Top Rated Bangalore Call Girls Mg Road ⟟ 8250192130 ⟟ Call Me For Genuine Sex...
Top Rated Bangalore Call Girls Mg Road ⟟ 8250192130 ⟟ Call Me For Genuine Sex...
 
(Rocky) Jaipur Call Girl - 09521753030 Escorts Service 50% Off with Cash ON D...
(Rocky) Jaipur Call Girl - 09521753030 Escorts Service 50% Off with Cash ON D...(Rocky) Jaipur Call Girl - 09521753030 Escorts Service 50% Off with Cash ON D...
(Rocky) Jaipur Call Girl - 09521753030 Escorts Service 50% Off with Cash ON D...
 

Air borne diseases

  • 1. AIR BORNE DISEASES & CASES APPLICATIONS By Doaa Habib Under supervision of Prof dr Mona Aboserea Zagazig university
  • 2. CONTENT  Infectious chain  Mode of transmission  Definitions  Classification  Viral infections  Emerging airborne infections  Bacterial infections  Questions
  • 3. INFECTIOUS CHAIN  Air borne or droplet or respiratory tract infections:  Agent : virus, bacteria, fungi.  Reservoir: mainly man, animal.  Exit: respiratory secretion.  Mode of transmission: inhalation  Inlet: nose, mouth.  Host: susceptible individual.
  • 4. MODE OF TRANSMISSION  Direct Droplet spread  Indirect Airborne Indirect contact
  • 5.
  • 6. DEFINITIONS  Air borne transmission :  occurs when infectious agents are carried by dust or droplet nuclei suspended in air.  Droplet nuclei are dried residue of less than 5 microns in size.  Droplet nuclei may remain suspended in the air for long periods of time and may be blown over great distances.  for example, Measles and TB .
  • 7. DEFINITIONS  Droplet spread :  Refers to spray with relatively large, short-range aerosols produced by sneezing, coughing, or even talking.  Droplet spread is classified as direct because transmission is by direct spray over a few feet, before the droplets fall to the ground.  Examples, Pertussis and meningococcal infection.
  • 9. CLASSIFICATION Less common infections: Viral: herpes simplex, Epstein-bar, hand, foot and mouth disease. Bacterial: chlamydia pneumonia, legionella, mycoplasma Fungal: Candida , Coccidioidomycosis, Cryptococcosis, Histoplasmosis, Pneumocystis carinii pneumonia.
  • 10. VIRAL: INFECTIOUS MONONUCLEOSIS AND HERPES SIMPLEX  Caused by Epstein Bar virus in case of infectious mononucleosis and Herpes simplex virus in case of Herpes simplex.  Infectious Mononucleosis is manifested by sore throat, lymphadenopathy, splenomegaly, skin rash and stomatitis.  Herpes Simplex usually produce vesicular stomatitis. The illness may starts acutely with fever and oral pain, followed by appearance of lesions on lips, gums, mucosa, tongue, palate. Management  Symptomatic  IV fluid in case of severe difficulty in swallowing.  Gentian violet helps to prevent secondary infection in Herpes simplex.  If secondary bacterial infection occurs, penicillin or metronidazole should be prescribed
  • 11. VIRAL: HAND, FOOT, AND MOUTH DISEASE  Hand, Foot, and Mouth Disease, or HFMD, is a viral contagious illness caused by coxsackie virus common in infants and children younger than 5 years old. Transmission: Close contact, droplet, contact with blister fluid  c/p: fever, mouth sores, and a rash, HFMD is usually not serious, and nearly all people recover in 7 to 10 days without medical treatment.  Complication: Rarely, viral meningitis polio-like paralysis, or encephalitis HFMD spreads from an infected person to others from: Management: symptomatic
  • 12. BACTERIAL: CHLAMYDIA PNEUMONIA  Agent: C. pneumoniae  IP: 3 to 4 weeks  Transmission: close contact , respiratory droplets, contact with contaminated surfaces.  c/p: cough, fever, headache, malaise and can develop:  Laryngitis, Pharyngitis, Pneumonia  Complication: Exacerbation of asthma, Encephalitis, Myocarditis  Diagnosis: 1. culture 2. Serology: CF, Micro immunofluorescence (MIF) is the serological method of choice 3. PCR  Management: 1) Macrolides (azithromycin) — first-line therapy 2) Tetracyclines (tetracycline and doxycycline): not for children 3) Fluoroquionolones
  • 13. LEGIONNAIRES’ DISEASE  Agent: Legionella bacteria (L. pneumophila)  Reservoir: natural, freshwater environments, human-made water systems (e.g., plumbing system of large buildings, cooling towers, decorative fountains, hot tubs)  IP: 3 to 4 weeks  Transmission: inhaling aerosolized water droplets, aspiration of contaminated drinking water. Legionella is usually not transmitted from person to person. however, a single episode of person-to person  Risk Factors  Age ≥50 years  Smoking (current or historical)  Chronic lung disease, such as emphysema or COPD  Immune system disorders due to disease or medication  Systemic malignancy  Underlying illness, such as diabetes, renal failure, or hepatic failure
  • 14. Legionnaires’ disease Pontiac fever Pontiac fever Clinical features Fever, myalgia, and cough shortness of breath, headache, confusion, nausea, diarrhea may be present flu-like illness, often with fever, chills, headache, myalgia, fatigue, malaise; less often with symptoms such as cough or nausea Pneumonia (clinical or radiographic) Yes No Pathogenesis Replication of organism Possibly an inflammatory response to endotoxin Incubation period 2 to 10** days after exposure 24 to 72 hours after exposure occuracce Less than 5% of people exposed to the source of Legionella Greater than 90% 3 Treatment Antibiotics Supportive care (because illness is self-limited) Isolation of the organism Possible Never demonstrated Outcome Hospitalization common Case-fatality rate: 10% (25% for healthcare-associated) Hospitalization uncommon Case fatality rate: extremely low
  • 15. LEGIONNAIRE DISEASE  Diagnosis: Best practice is to obtain both sputum culture and the urinary antigen test Indications for Legionnaires’ Disease Testing:  Patients who have failed outpatient antibiotic treatment for community-acquired pneumonia  Patients with severe pneumonia, in particular those requiring intensive care  Immunocompromised patients with pneumonia*  Patients with a travel history (patients who have traveled away from their home within 10 days before the onset of illness)  All patients with pneumonia in the setting of a Legionnaires’ disease outbreak  Patients at risk for Legionnaires’ disease with healthcare-associated pneumonia (pneumonia with onset ≥ 481 hours after admission)  Management: Macrolides and respiratory fluoroquinolones
  • 16. MYCOPLASMA PNEUMONIA  Agent: M. pneumoniae  IP: 1 to 4 weeks  Transmission: airborne droplets from person to person and is exclusively a human pathogen.  c/p: cough, fever, headache, malaise and can develop: Tracheobronchitis, Pharyngitis Pneumonia  Complication: Exacerbation of asthma, Encephalitis, Myocarditis, Hemolytic anemia, Renal dysfunction, Erythema multiforme, Stevens- Johnson syndrome, or toxic epidermal necrolysis  Diagnosis: 1. culture 2. Serology: enzyme immunoassay (EIA) testing 3. PCR  Management: it does not respond to sulfonamides or penicillin (beta- lactams). 1) Macrolides (azithromycin) — first-line therapy 2) Tetracyclines (tetracycline and doxycycline): not for children 3) Fluoroquionolones
  • 17. FUNGAL INFECTION Candidiasis This illness is caused by Candida. Candidiasis, can affect the skin, nails, and mucous membranes throughout the body. Persons with HIV infection often have trouble with Candida, especially in the mouth and vagina. However, candidiasis is only considered an OI when it infects the esophagus (swallowing tube) or lower respiratory tract, such as the trachea and bronchi (breathing tube), or deeper lung tissue. Coccidioidom ycosis This illness is caused by the fungus Coccidioides immitis. It most commonly acquired by inhaling fungal spores, which can lead to a pneumonia that is sometimes called desert fever, San Joaquin Valley fever, or valley fever. Cryptococcos is This illness is caused by infection with the fungus Cryptococcus neoformans. The fungus typically enters the body through the lungs and can cause pneumonia. Histoplasmosi s This illness is caused by the fungus Histoplasma capsulatum. Histoplasma most often infects the lungs and produces symptoms that are similar to those of influenza or pneumonia. People with severely damaged immune systems can get a very serious form of the disease called progressive disseminated histoplasmosis.
  • 18. PNEUMOCYSTIS PNEUMONIA  Agent: fungus Pneumocystis jirovecii (carinii)  IP: 1 to 4 weeks  Risk: HIV, Solid organ transplant, Blood cancer, Inflammatory diseases or autoimmune diseases, Stem cell transplant  Transmission: airborne droplets from case or carrier.  c/p: cough, fever, headache, Difficulty breathing, Chest pain, Chills Pneumonia  Complication: highly fatal.  Diagnosis: 1. sputum or bronchoalveolar lavage Or lung tissue biopsy is examination under a microscope. 2. PCR 3. A blood test to detect β-D-glucan  Management: trimethoprim/sulfamethoxazole also known as co-trimoxazole  Prevention: co-trimoxazole for patients at risk.
  • 19. German measles “Rubella” Measles “Rubeola” Cases & incubatory Carriers. Congenitally Infected infants act as reservoir for 1 year"in respiratory secretion, blood, urine, & stool” Man only case or contact healthy carrier (inresp. secretion) Source of infection 14-18 days10 daysIP 1 week before & 7 days after rash In prodromal stage & 5 days after rash Period of infectivity
  • 20. German measles “Rubella” Measles “Rubeola” C/P - 1 day - Mild symptom - Cervical lymph adenopathy. - 3-4 days Fever, cough, Catarrh, Conjunctivitis - Koplik's spots on 2nd day “Tiny bluish white spots on buccal mucosa”. Prodroma lstage - Lasts for 3 days. - Maculo-papular or uniform red all over body. On 4th day red blotchy appears on face, root of hair then generalized. Branny desquamation after 1 week. Rash - Congenital rubella syndrome “CRS”: infection during 1st trimester(16 weeks). Causes cataract, deafness, heart anomaly, mental retardation. - Otitis media, cervical adenitis - Encephalitis - Pneumonia - Otitis media Complication
  • 21. German measles “Rubella” Measles “Rubeola” Pre vent ion Rubella alone or MMR live attenuated, single dose 0.5ml SC. given at 12-15th month. Given to adolescent girls or females at any age before pregnancy. MMRV:12M - 12 years of age. The minimum interval is 3 months Life long immunity. Type: live attenuated virus Preparations: MMR, MMRV Administration: 0.5 ML subcutaneous injection. Schedule: MMR: 2 doses, 1st at 12-15 months and booster dose( at least 28 day after 1st dose) at 4-6 years before school entry. Life long immunity Vaccination Vaccine and IGS are not effective Pregnant exposed to infection & refuse abortion. Vaccine within 3days of exposure is protective Within 6 days of exposure Sero-prevention: Human Ig. Sero-attenuation: smaller dose to give mild C/P. PEP
  • 22.
  • 23.
  • 24. ZosterChicken poxSmall pox Preparation, type: Zostavax: live attenuated virus. Shingrix :recombinant zoster vaccine Administration: 0.5ml SC injection in arm Schedule: Zostavax: single dose. Shingrix : IM two doses separated by 2 to 6months. Type: live attenuated virus Preparation: varivax (varicella alone), MMRV Administration: 0.5 ML subcutaneous injection Schedule: First dose at 12 -15 months old, Second dose at 4 -6 years Calf lymph vaccine: vaccinia virus ACAM2000 Aventis Pasteur Smallpox Vaccine (APSV): Imvamune: attenuated live vaccine, underdevelopment. Administration, Schedule : Calf lymph vaccine: single dose by scratching. ACAM2000, Aventis Pasteur Smallpox Vaccine (APSV): single dose by the percutaneous route using the multiple puncture technique. Vaccination varicella vaccine: within 3 daysup to 5 days, of exposure to rash. zoster immune globulin (ZIG): within 72 to 96 hours after exposure Vaccination Within 3 days prevent, 5-7 days mild symptoms PEP
  • 25. VIRAL INFECTIONS WITHOUT RASH MumpsCommon coldinfluenza VirusRhino v.Virus types A:mutation &pandemics B:epidemic C:outbreaks Agent Cases & incubatory carriers (saliva & in utero) CasesHuman cases only “may be Avian, swine, horses” Source of infection 18 days1-3 daysIP 1 week before parotitis till disappearance of swelling. All course of diseasePeriod of infectivity
  • 26. MumpsCommon cold influenza Prodroma: 1-2 days fever, malaise, headache, body aches. Enlarged painful salivary gland (parotid, sublingual, submaxillary) Sudden onset catarrh, running nose, sneezing, sore throat, cough. Usually no fever Sudden high fever, body aches, arthralgia, sore throat, cough. C/P Orchitis, oophritis, aseptic meningitis, pancreatitis, mastitis, nephritis, neuritis Rare.Pneumonia, otitis media, sinusitis, pericarditis, bronchitis Complication
  • 27. Mumps C.C influenzaVaccination MMR 0.5ml SC or mumps vaccine alone. Must be before puberty. Life long immunity. No 1- Inactivated : Trivalent: 5 years – 65 years, high dose, adjuvant vaccine > 65 years Quadrivalent: 6months – 64 years 2- Live attenuated: Trivalent, Quadrivalent: 2 years – 64 years 3- Quadrivalent Recombinant: > 18 years Administration, Schedule : Inactivated:0.5 ml IM, 2 doses separated by 4 weeks. Live attenuated: 0.2 ml intranasal, one dose. PEP booster dose of MMR within 5 days Chemoprophylaxis: Oseltamivir, Zanamivir
  • 28. EMERGING VIRAL AIR BORNE INFECTIONS  Avian flu( H5N1)  Swine flu (H1N1)  SARS  MERS- COV
  • 29. Swine fluAvian flu H1N1H5N1, H7N9(Asian form) Ag en t pigsPoultry, wild birds, pigs S. INF 2 - 14 days (internationally 7 days) 14 days (internationally 7 days) IP All coarse of diseaseMan to man transmission Is very rare infec tivity mild to severe and included conjunctivitis, influenza-like illness (e.g., fever, cough, sore throat, muscle aches), pneumonia sometimes accompanied by nausea, abdominal pain, diarrhea, and vomiting C/P Fatality is lowacute respiratory distress, respiratory failure), neurologic changes (altered mental status, seizures), and the involvement of other organ systems. Complicati on The H1N1 Inactivated Vaccine (2 dose, IM) The H1N1 Live Attenuated Intranasal Vaccine Vaccine for birds Vaccination Like flu P E P
  • 31.
  • 32. Scarlet feverStrpt. Pharyngi tis Diphteria NoUnder trial Type: Diphtheria: toxoid Preparation: 1- Diphtheria and Tetanus (DT and Td) 2- Diphtheria, Tetanus, acellular Pertussis (DTaP) Vaccines 3- Tetanus, Diphtheria, acellular Pertussis (Tdap) Vaccines IM injection on 2, 4, 6 months, 15 -18 months, and 4 through 6 years Vaccination Chemoprophylaxis is not recommended Antibiotics: benzathine penicillin G, erythromycin. Diphtheria antitoxin is not indicated for prophylaxis of contacts of diphtheria patients. PEP
  • 33.
  • 34. TBPertussisMeningitis Preparation, type: Live attenuated bovine strain. Administration, Schedule : 0.1 ml intradermal injection in deltoid of lt arm, single dose Type: Pertussis; killed bacteria in DPT Preparation: 1- Diphtheria and Tetanus (DT and Td) 2- Diphtheria, Tetanus, acellular Pertussis (DTaP) Vaccines 3- Tetanus, Diphtheria, acellular Pertussis (Tdap) Vaccines IM injection on 2, 4, 6 months, 15 -18 months, and 4 through 6 years Capsular polysaccharide vaccine: quadrivalent, capsular polysaccharide Meningococcal conjugate: quadrivalent, meningococcal A, C, W, and Y polysaccharides conjugated to protein adjuvant. serogroup B meningococcal vaccines: monovalent, recombinant protein vaccines Administration, Schedule : Capsular polysaccharide vaccine: 0.5 ml IM, 2 doses 3months apart Meningococcal conjugate: 0.5 ml IM, 2 doses 2 months apart serogroup B meningococcal vaccines:0.5 ml IM, 2 doses 1months apart Vaccination INH orally for 1 year .Oral erythromycin in 4 divided doses for 10 days Rifampin 600mg twice 2 days International certificate to endemic area, ciprofloxacin, and ceftriaxone may be used PEP
  • 35. DPT VACCINE Preparation: 1- for children < 7 years (DTaP and DT) 2- for persons > 7 years (Tdap and Td) They differ in concentrations of diphtheria and tetanus concentrations 1- for children < 7 years: concentration of DT toxoid (25 Lf of diphtheria toxoid, 10 Lf of tetanus toxoid) is higher than td and D toxoid concentration is higher than T toxoid. 2- for persons > 7 years: concentration of td toxoid (5 Lf tetanus toxoid, 2 Lf diphtheria toxoid) is lower than TD and t toxoid concentration is higher than d toxoid. Contraindications to acellular pertussis-containing vaccines: 1. Patients who developed encephalopathy within 7 days of administration of a previous dose of DTP, DTaP, or Tdap 2. Progressive or unstable neurologic disorder (including infantile spasms for DTaP) 3. Uncontrolled seizures 4. Progressive encephalopathy 5. Had Guillain-Barré syndrome within 6 weeks after a previous dose of tetanus toxoid-containing vaccine
  • 36.
  • 37. Pneumococcal diseaseHib disease lancet-shaped, gram-positive capsulatedgram-negative coccobacillus. Mostly encapsulated Ag nasopharyngeal carrier or case patientnasopharyngeal carrier or case patient. aspiration of amniotic fluid or contact with genital tract secretions S.inf 1 to 3 days2-4 days Ip transmission can occur as long as the organism appears in respiratory secretions. Secondary attack rate is 2-6% p.If Ear infections, Sinus infections, Pneumonia, Meningitis, Bacteremia Pneumonia, Bacteremia, Meningitis, Epiglotittis, Cellulitis, Infectious arthritis C/P Preparation, type: pneumococcal conjugate vaccine (PCV7): IM Pneumococcal Conjugate Vaccine (PCV13); Pneumococcal Polysaccharide Vaccine (PPSV23): IM or SC Administration, Schedule: (2 doses, 8 weeks apart) Routine: at 2, 4, and 6 months of age, Booster dose at 12 through 15 months of age. Preparation, type: a polysaccharide conjugate vaccine. Monovalent: Hib Combined: pentacel ( DTaP + Hib + IPV) Administration, Schedule: IM injection at 2,4,6 months OR 2,4 months and booster 12 - 15 months Vacc Not recommendedRifampicin 600mg twice daily pep
  • 38. CASE STUDY A "5" years old girl was bought to the outpatient clinic because of rash covering the girl's face, it was preceded by fever, malaise for 3 days. a)What is the differential diagnosis? On examination, the rash was red blotchy maculopapular rash and there was bluish spot on the girl’s buccal mucosa. b) What is the provisional diagnosis? c) How to confirm? d)What are the preventive and control measures that should be done for the case, house hold contact including 10 months brother and her school contacts .
  • 39. CASE 1 a) Differential diagnosis:  Measles (maculopapular, first on face, on 4th day of high fever)  Rubella (maculopapular, after 1 day of low fever , cervical lymph nodes)  Varicella ( pleomorphic, after 2-3 days of low fever, first on trunk centripetal)  smallpox ( monomorphic, high fever, first on extremities) Less likely  Scarlet fever (no rash in the face)  Meningitis (high fever, convulsions)  Drug or food allergy (history of exposure)
  • 40. CASE 1 b) the provisional diagnosis: probable case of measles c) Confirm diagnosis:  Isolation of measles virus from a clinical specimen; or  Detection of measles-virus specific nucleic acid from a clinical specimen using polymerase chain reaction; or  IgG seroconversion or a significant rise in measles immunoglobulin G antibody using any evaluated and validated method; or  A positive serologic test for measles immunoglobulin M antibody; or  Direct epidemiologic linkage to a case confirmed by one of the methods above.
  • 41. CASE 1 Case: notification to LHU- isolation (4 days after rash in measles)- concurrent disinfection-ttt- release after rash disappearance. Treatment  There is no specific antiviral therapy. Symptomatic, address complications such as bacterial infections.  Severe measles cases among children should be treated with vitamin A. Contact: enlistment- surveillance - HE Segregation: from school, child care, health care setting MMR vaccine: within 72 hours of initial measles exposure to those without evidence of immunity Immunoglobulin (IG) : People who are at risk for severe illness and complications from measles, such as infants younger than 12 months of age, pregnant women and immuno compromised people Its recommended to give vaccine rather than IGS to the baby and later on he take the routine 2 doses of MMR School contacts If sure of immunization (document 2 doses): survillance- HE to avoid further exposure. If not sure of previous immunization, or child have fever or respiratory catarrah before or the child is immunocompromised: seroprevention, seroattenuation Outbreak measures: booster dose MMR
  • 42. CASE 2 A student 7 years old complaining of fever, sore throat, headache and malaise was detected by school nurse. a)What is the provisional diagnosis? b)What is the differential diagnosis? c)What are the preventive measures that should be done in school according to the type of disease identified? d)What are the measures taken for the sick child? His house hold contacts?
  • 43. CASE 2  provisional diagnosis: Streptococcal pharyngitis.  differential diagnosis:  Strept. Pharyngitis  Diphtheria  Influenza :short I.P(1-3 days), sudden onset of fever, constitutional manifestation  Common cold: mild fever, running nose  Less likely: Meningitis: high fever, convulsions  Prodroma of measles, rubella
  • 44.
  • 45. CASE 2 preventive Measures done in school General preventive measures: good ventilation, HE, notification if get ill In strept: chemoprophylaxis is not recommended In diphtheria: close survillence and administer antitoxin with the first sign of disease.  if immunized before: booster dose of D toxoid, not immunized 2 doses of vaccine.  prophylaxis by antibiotics to all contacts  benzathine penicillin G Single dose. or  a 7- to 10-day course of oral erythromycin (40 mg/kg/day for children and 1 g/day for adults).
  • 46. CASE 2 measures taken for the sick child? His house contact ? Child: Penicillin or amoxicillin is the antibiotic of choice to treat group A strep pharyngitis. for 10 days Diphtheria:  Respiratory support and airway maintenance should also be administered as needed. Diphtheria Antitoxin  Diphtheria antitoxin, 20,000 - 80,000 I.U. (or even 100,000 in severe cases) I.M. or I.V.  The patient must be tested for sensitivity before antitoxin is given. Antibiotics: The antibiotics of choice are  erythromycin (500 mg four times daily for 14 days) or  procaine penicillin G (600,000 units every 12 hours intramuscularly) until the patient can take oral medicine, followed by oral penicillin V (250 mg four times daily) for a total treatment course of 14 days.  Elimination of the organism should be documented by two consecutive negative cultures after therapy is completed at least 24 hours apart .  Maintain isolation until elimination of the organism .  Vaccination is required because clinical diphtheria does not necessarily confer immunity
  • 47. CASE 2 House contacts: Strept. Pharyngitis: HE, SURVIELLENCE FOR 3 DAYS Diphtheria:  close surveillance and begin antitoxin at the first signs of illness.  a diphtheria booster if vaccinated before, 2 doses if not vaccinated. AND  Antibiotics:  benzathine penicillin G Single dose (600,000 units for persons younger than 6 years old and 1,200,000 units for those 6 years old and older) or  a 7- to 10-day course of oral erythromycin (40 mg/kg/day for children and 1 g/day for adults).  Identified carriers should also receive antibiotics.  Adult with occupational contact with children —» allow to go to work only after 2 more -ve swabs.  If the swab is positive —» Segregate (exclude) from school or work until 3 consecutive negative swabs 24 hours apart is obtained.  Diphtheria antitoxin is not indicated for prophylaxis of contacts of diphtheria patients
  • 48. CASE 3 A male adult aged 50 years old working as public bus driver, complaining of low fever, loss of weight, night sweating, cough and anorexia. Symptoms began since 2 weeks. a) What is the provisional diagnosis? b) How can you verify your diagnosis? c) What are the preventive measures that must be done for that case, wife, 1 and 18 years old children and his old father 70 years?
  • 49. CASE 3 the provisional diagnosis: Probable Pulmonary tuberculosis - history: elderly-male patient - c/p: low grade fever, loss of wt, anorexia, night sweating, cough Confirm diagnosis: A) Bacteriology: 1) Detection of TB bacilli ( Direct smear microscopy, laryngeal swabs, culture). 2) Tuberculin test ( immune response to TB). 3) Histo-pathological diagnosis. 4) BACTEC ( detection of metabolic end products of bacilli). 5) PCR ( detection of DNA of TB bacilli). B) Radiology: sensitive but not specific
  • 50. CASE 3 Measures for case: case finding, notification to LHU, isolation for 4-8 ws, disinfection, ttt, follow 5 years Treatment: DOTS  first 2 months: isoniazid + rifampicin + pyrazinamide + ethambutol  following 4 months: isoniazid + rifampicin Measures for Contacts: enlistment- HE  tuberculin testing  tuberculin -ve give BCG vaccine and prevent contact with the case till 3 months  tuberculin +ve do chest x ray, if suspicious do smear exam. Give INH. Chemoprophylaxis: INH (isonicotinic acid hydrazid) orally for 1 year. Year: BCG in first 3 months, so TST is +ve > 15mm Sputum exam (+ve: DOTS), (-Ve: chemoprophylaxis) 18 years: TST test, sputum exam If both +ve (DOTS)- IF both –ve ( BCG) IF TB +ve and sputum –ve: chemoprophylaxis Father (70 ys), Pregnant: immunocompromised C.I of BCG>35ys, INH risk of induce hepatitis
  • 51. CASE 4 A 15 years old girl came to the out patient clinic complaining from high fever, vomiting, neck rigidity and convulsion. a) What is the differential diagnosis? On examination there was dark red petechial eruption on extremities. b) What is the provisional diagnosis? c) How to confirm? d) What is the measures taken to the case and her contacts?
  • 52. CASE 4 a) Meningitis for differential diagnosis Bacterial:  Neisseria meningitides  haemophilus influenza  Streptococcus pneumoniae  Group B Streptococcus  Listeria monocytogenes Viral:  Non-polio enteroviruses, the most common  Mumps virus  Herpesviruses, including , herpes simplex viruses, and varicella- zoster virus.  Measles virus  Influenza virus  Arboviruses, such as West Nile virus Fungal: in immunocompromised Cryptococcus, Histoplasma Non infectious: autoimmune, injury, tumors
  • 54. CASE 4 b) Provisional diagnosis: suspected meningococcal meningitis due to presence of purpura fulminans. c) Confirm diagnosis:  Detection of N. meningitidis-specific nucleic acid in blood or CSF), using a valudated polymerase chain reaction (PCR) assay; or  Isolation of N. meningitidis From blood or CSF, or less commonly or purpuric lesions.
  • 55. CASE 4 d) Measures to case, contact. Case:  effective antibiotics should be administered promptly to patients suspected of having meningococcal disease.  treatment with penicillin G, ampicillin, or an extended-spectrum cephalosporin (cefotaxime or ceftriaxone) .  if antimicrobial agents other than ceftriaxone or cefotaxime are used for treatment, eradication of nasopharyngeal carriage with rifampin (4 doses over 2 days) or single doses of ciprofloxacin or ceftriaxone are recommended prior to discharge from the hospital. Contact:  Antibiotic chemoprophylaxis: ideally should be initiated within 24 hours after the index patient is identified; prophylaxis given >2 weeks after exposure has little value.  Antibiotics used ciprofloxacin, rifampin, and ceftriaxone.  Ceftriaxone is recommended for pregnant women.  Rifampin 600mg twice daily.
  • 56. CASE 5 A 35 years old man come to out patient clinic complaining from chest pain, productive Cough, Dyspnea and high fever The patient is smoker for 5 years. a) What is provisional diagnosis? b) What is the causative agent of the disease? c) The gram stain shows gram positive cocci, How to confirm diagnosis? d) What are measures taken for the patient and his contacts?
  • 57. CASE 5 a) provisional diagnosis: pneumonia b) Causative organism: Viral: influenza and respiratory syncytial virus (RSV) Bacterial: pneumococcus, legionella, mycoplasma, H. influenza Fungal: pneumocystis pneumonia. c) pneumococcal pneumonia confirmed by:  Supportive: Identification of S. pneumonia from a normally sterile body site by a CIDT(culture independent diagnostic tests like PCR ) without isolation of the bacteria.  Detect capsular polysaccharide antigen in body fluids or in urine by immunochromatographic membrane technique.  Confirmatory: Isolation of S. pneumonia from a normally sterile body site by culture.
  • 58. CASE 5 d) Measures taken for the patient and his contacts: Patient: notification, isolation, disinfection, treatment General: support respiration, symptomatic. Specific: S. pneumonia are resistant to one or more clinically relevant antibiotics so treatment depend on culuture and sensitivity. Contacts: enlistment, HE, surveillance for 3 days Chemoprophylaxis The American Academy of Pediatrics typically recommends daily antimicrobial prophylaxis with oral penicillin V or G for children with functional or anatomic asplenia, especially those with sickle-cell disease. Because secondary cases of invasive pneumococcal infection are uncommon, chemoprophylaxis is not indicated for contacts of patients with such infection. Vaccination: pneumococcal conjugate vaccine (PCV13), pneumococcal polysaccharide vaccine (PPSV23)  Are recommended for all children, old age, adults with certain diseases  It is also recommended after infection because infection doesnot prevent future attacks.
  • 59. CASE 6 5 years girl was brought to your office by her mother complaining from pruritic vesicular rash covering her body and preceded by fever. a) What is your diagnosis? b) How to confirm it? c) What are the measures taken for the girl and her contacts?
  • 60. CASE 6 a) Diagnosis: probable case of chicken pox (varicella infection) b) Confirm diagnosis:  Epidemiologic linkage to another probable or confirmed case, OR  Laboratory confirmation by any of the following:  Isolation of varicella virus from a clinical specimen, OR  Varicella antigen detected by direct fluorescent antibody test, OR  Varicella-specific nucleic acid detected by polymerase chain reaction (PCR), OR  Significant rise in serum anti-varicella immunoglobulin G (IgG) antibody level by any standard serologic assay
  • 61. CASE 6 c) Measures to girl & contacts : The girl: notification, isolation 7 days, disinfection ,ttt  symptomatic, analgesics, calamine lotion and antihistaminic  treatment with antivirals is not routinely recommended for otherwise healthy children with varicella. Contacts: Previous Varicella infection or 2 doses of vaccine give life long immunity, no need to PEP. varicella vaccine: is effective in preventing illness or modifying varicella severity if administered to unvaccinated children within 3 days, and possibly up to 5 days, of exposure to rash. zoster immune globulin (ZIG): within 72 hours of exposure , 96 hours after exposure in Immuno compromised patients but may be effective if administered as late as 10 days .
  • 62. CASE 7 11 years old boy come to your office complaining from fever and swelling behind his ear. a) What is the possible diagnosis? b) What are the complications of such case? c) What are the measures taken for the boy and his contacts?
  • 63. CASE 7 a) Diagnosis: suspected mumps case b) Complication :  Orchitis, usually unilateral in 20-30% of post pubertal males (rarely may induce sterility)  CNS involvement (aseptic meningitis, Encephalitis).  Pancreatitis.  Neuritis, arthritis, mastitis, nephritis, thyroiditis and pericarditis may occur.  Permanent nerve deafness (usually unilateral) is a rare complication.
  • 64. CASE 7 c) Measures for the boy & contacts: The boy: notification, isolation, disinfection, ttt. Isolation: relieve of swelling (9 days). Treatment: symptomatic Contacts: enlistment, segregation, HE, surveillance for 2 weeks.  2nd dose of MMR within 5 days.  third MMR dose administered as PEP did not have a significant effect, it may offer some benefits in specific outbreak contexts.  Immune globulin (IG) is not effective postexposure prophylaxis.
  • 65. CASE 8 10 months baby boy brought by his mother to your office complaining from attacks of productive cough which increase at night and end with vomiting. The mother also complained from strange cough sound. a) What is the provisional diagnosis? b) What are the complication of this disease? c) What are the measures taken to the baby and his contacts?
  • 66. CASE 8 a) Diagnosis: whooping cough ( pertussis infection). b) Complications: 1- Increasing pressure during paroxysmal coughing: * Hernia (especially umbilical) and prolapse of rectum. * Convulsions in infants due to cerebral anoxia. * Hemorrhage: skin, nasal, conjunctiva and C.N.S. *Encephalopathy in severe cases from cerebral anoxia or hemorrhage. 2- Secondary bacterial infection: may cause otitis media, diarrheal disease, pneumonia and bronchopneumonia. 3- Malnutrition: due to repeated vomiting for long time in untreated cases. 4- Long term complications which include neurological deficits ranging from gross mental retardation to behavior disorders.
  • 67. CASE 8 c) Measures for baby & contact: Baby: Case finding, Notification, Isolation , disinfection. Treatment:  Specific: - erythromycin : administered in 4 divided daily doses for 14 days  symptomatic: - Proper feeding. - Prevention and management of complications. - Release: pupils can return to school after 3 weeks from start of whooping stage and improvement of the case clinically or one week from beginning of effective treatment. Contacts: enlistment, HE, surveillance 2 weeks Chemoprophylaxis: Oral erythromycin 50 mg/kg/day in 4 divided doses for 10 days. Providing PEP to all household contacts of a pertussis case.
  • 68. CASE 9 65 years old man visit the out patient clinic complaining from painful rash on his shoulder for 3 days? a) What is the provisional diagnosis? b) What is the complication? c) What are the measures taken for the patient and his contacts, wife, pregnant daughter , and 10 years grandson?
  • 69. CASE 9 a) Diagnosis: zoster infection (VZV ) b) Complications:  post-herpetic neuralgia  cranial or peripheral nerve palsies  sensory loss, deafness  ocular complications.  bacterial infection of the lesions, usually due to Staphylococcus aureus.  visceral involvement, meningoencephalitis, pneumonitis, hepatitis, and acute retinal necrosis.
  • 70. CASE 9 c) Patient: notification, isolation, disinfection  Antiviral drugs: acyclovir, valacyclovir, and famciclovir  Analgesics  Wet compresses, calamine lotion to relieve some of the itching.  Vaccination (Shingrix ) after acute attack to prevent coming episodes. Contacts: enlistment, HE, surveillance Wife:  Evidence of varicella immunity: nothing  No Evidence of immunity and immunocompromised: VZIG during 72 – 96 Hours OR acyclovir for 7 days after 7 -10 days post exposure.  Shingrix should be offered
  • 71. CASE 9 Pregnant daughter:  Evidence of varicella immunity: nothing  No Evidence of immunity:  VZIG during 72 – 96 Hours.  if she has signs and symptoms of varicella around the time of delivery 5 days before to 2 days after delivery, VZIG should be given to her neonate. 10 years grandson:  Evidence of varicella immunity: nothing  No Evidence of immunity: varicella vaccine: within 3 days, and possibly up to 5 days, of exposure to rash. zoster immune globulin (ZIG): within 72 hours of exposure , 96 hours after exposure in Immuno compromised patients
  • 72. CASE 10 6 months baby girl was brought to the out patient clinic suffering of fever, running nose and fast breathing .Her mother has fever, body aches, cough and sore throat for 2 days. a) What is the differential diagnosis? b) How to confirm diagnosis? c) What are the measures taken for the girl and her contacts?
  • 74. CASE 10 Other less possible diagnosis: 1. otitis media: fever, ear pain, irritability, crying. 2. sinusitis: postnasal discharge, high fever, cough, headache. 3. Strept. Pharyngitis: high fever- sore throat-red oedmatous pharynx, tonsil 4. Catarrhal stage of measles, rubella 5. Pertussis: upper respiratory catarrh for 10 days, paroxysmal attack of spasmodic cough (whooping cough) b) Confirm diagnosis: Clinically, the girl has influenza infection and early sign of pneumonia The diagnosis base on symptoms, history of contact with flu patient and fast breathing as early sign of pneumonia. o A case definition of fever 100°F or greater and cough and/or sore throat is used by CDC in Influenza-like Illness Surveillance Network (ILINet). o Individual cases of influenza typically are not investigated. Exceptions to this are severe, unusual complications. Or o suspected or confirmed to be of animal origin (most frequently swine or avian). o Definitive diagnosis of influenza requires laboratory confirmation in addition to signs and symptoms.
  • 75. CASE 10 Measures to girl & contacts: Girl: notification, isolation, disinfection, ttt Treatment: Oseltamivir 3mg/kg twice daily for 5 days. Contacts: Chemo prophylaxis:  Antiviral chemoprophylaxis generally is not recommended if more than 48 hours have elapsed since the first exposure to a person with influenza.  once daily for 7 -14 days  Oseltamivir:75 mg, oral, used for 3months and older.  Zanamivir: 10 mg, inhaled , recommended from age 7 years and older. Indication:  people at high risk for complications who cannot receive influenza vaccine.  people at high risk of influenza complications during the first two weeks following vaccination.  people with severe immune deficiencies or others who might not respond to influenza vaccination. 2- vaccine: no post exposure prevention but still recommended if still in flu season.
  • 76. CASE 11 3 years old boy visited the outpatient clinic with his mother complaining from generalized rash first appear on face and preceded by mild fever, on examination there was enlarged cervical lymph node. a) What is the possible diagnosis? b) How to confirm? c) What are measures taken for the child and house hold contacts including pregnant sister?
  • 77. CASE 11 a) diagnosis: probable rubella infection b) Confirm diagnosis:  Isolation of rubella virus; or  Detection of rubella-virus specific nucleic acid by polymerase chain reaction; or  IgG seroconversion or a significant rise between acute- and convalescent-phase titers in serum rubella IgG antibody level by any standard serologic assay; or  Positive serologic test for rubella IgM antibody†* OR  Epidemiologic linkage to a laboratory-confirmed case of rubella.
  • 78. CASE 11 Case: Isolation: for 5--7 days after rash onset. Treatment : There is no specific antiviral therapy. Symptomatic, address complications such as bacterial infections. Contact: Rubella vaccine and IG are not effective as PEP MEASURES taken to prevent spread of infection:  Identify and vaccinate susceptible persons who have no contraindications to rubella vaccine.  Ensure that pregnant women who are exposed to rubella are serologically evaluated for rubella-specific IgM and IgG antibodies.  Infection indicated by +ve IgM or rising titre of IgG or IgG seroconversion.  Counsel susceptible pregnant women regarding the risks for intrauterine rubella infection and recommend that they restrict their contact with persons with confirmed, probable, or suspected rubella for >6 weeks (two incubation periods) after rash onset in the last identified patient.  Abortion is recommended, IG in high dose if the mother refuse abortion.
  • 79. MCQ There are reports of an outbreak of mumps in your community. As you prepare to see patients who may be infected, what should you consider regarding mumps? A. Two doses of the measles-mumps-rubella (MMR) vaccine provides over 99% protection against mumps B. Mumps is only communicable after the onset of parotitis C. Up to one-quarter of individuals infected with mumps virus may be asymptomatic D. Public health policies in schools effectively eliminate the risk of the spread of mumps Key: C
  • 80. MCQ 2. As regard a third dose of MMR as post exposure prophylaxis. What is the policy of the Advisory Committee on Immunization Practices (ACIP)? A. It should be administered to household contacts only B. It should be administered to children between the ages of 5 and 12 years only C. It should be applied broadly in affected communities D. It should not be used at all KEY: A
  • 81. MCQ What was the main result of using 3rd dose MMR as post exposure prophylaxis against mumps? A. MMR was not associated with a numerical or statistical benefit B. MMR was associated with a lower rate of secondary cases of mumps, but the result was not statistically significant C. MMR significantly reduced the risk of secondary cases of mumps D. MMR caused more cases of mumps than it prevented Key: B
  • 82. MCQ 4. The commonest cause of bacterial meningitis in newborns is: A. Group B Streptococcus B. Streptococcus pneumoniae C. Listeria monocytogenes D. Escherichia coli KEY: A
  • 83. MCQ 4. The commonest cause of bacterial meningitis in children is: A. Streptococcus pneumoniae B. Neisseria meningitidis C. Haemophilus influenzae type b (Hib) D. group B Streptococcus KEY: A
  • 84. MCQ The most incriminated organism in causing bacterial meningitis in Teens and young adults is: A. Neisseria meningitidis B. Streptococcus pneumoniae C. Listeria monocytogenes D. Haemophilus influenzae type b (Hib) KEY: A
  • 85. MCQ The main causative organism of viral meningitis is: A. Non polio enterovirus B. Mumps virus C. Measles virus D. Influenza virus KEY: A
  • 86. MCQ A presumptive case of pulmonary TB in which there is only 2 of 3 smears are positive for AFB. What is the most appropriate action? A. Do a third confirmatory smear B. Initiate treatment C. Ask for x ray D. Ask for TB culture KEY: B
  • 87. MCQ 50 years old man complain from cough for 1 month, anorexia, night sweat and fatigue. On sputum analysis, only one of three smears was positive for AFB, what is the suitable action? A. Repeat sputum analysis B. Begin treatment immediately C. Ask for x ray D. Consider another diagnosis Key: c
  • 88. MCQ Tuberculin test is considered positive in previously immunized person with BCG if induration is: A. ≥ 5 mm B. ≥ 10 mm C. ≥ 15 mm D. 10 – 15 mm Key: c
  • 89. MCQ All the following statements are wrong about tuberculin test results except: A. Indurations of ≥ 15 mm is positive in HIV positive non immunized patient B. Indurations of ≥ 15 mm is positive in healthy non immunized individual. C. Indurations of ≥ 10 mm is positive in children before age of 5 years D. Indurations of ≥ 10mm is positive in children after age of 6 years Key: D
  • 90. MCQ A 5 year old child comes to the immunization centre without BCG scar on his arm; what would you prefer? a) Give BCG vaccine b) Perform mantoux if positive then give BCG c) No need of BCG d) Chemoprophylaxis e) Perform mantoux if negative then given BCG Key: e
  • 91. A mother brought her six weeks old child to an EPI centre for routine immunization. She was enquired about history of Epilepsy in the family and febrile fits. The doctor took this history to avoid complication with: a) Diphtheria toxoid b) Tetanus toxoid c) Hepatitis B vaccine d) Pertussis vaccine e) OPV Key: True: d
  • 92. A 6 weeks old boy came for DPT, polio & HBV vaccination. He was given initial doses of all and was called after 4 weeks to have the next doses. The likely reason for calling him again was: a) Loss of immune memory b) Stimulation of macrophages c) Summation of immune responses d) Replication of lymphocytes e) Immune tolerance Key: True: c
  • 93. A mother brought her four year old child to the doctor. She gave the history that her child was in close contact with a case of diphtheria in school. She was very anxious about her child and gave history of booster dose of DT 2 years ago. What would be line of management for such a child? a) Booster dose of DT with penicillin b) Active and passive immunization c) Active and passive immunization with chemoprophylaxis d) Only keep under surveillance for 1 week e) Nothing more required Key: a
  • 94. In Sir-Syed Model School a student of class 3 developed measles. The child was isolated from rest of the class. The school medical officer advised for the contacts of this child: a) Active immunization within 3 days b) Passive immunization c) Chemoprophylaxis d) Isolation e) Anti-viral therapy Key: b
  • 95. A woman brought her child with congenital anomalies of heart and cataract. She gave history of mild fever and rash in the first trimester of pregnancy, which settled with mild antipyretics. The possible gestational condition that resulted in these anomalies was: a) Measles b) Herpes c) Rubella d) Drugs taken in pregnancy e) Streptococcal scarlet fever Key: True: c
  • 96. A 22 years old man presented with painful, vesicular and postulate eruption on his back. He gave history of chicken pox infection at ten years of age. The likely diagnosis was: a) Measles b) Meningococcemia c) Herpes simplex d) Scarlet fever e) Herpes zoster Key: True: e
  • 97. There is an epidemic of Meningococcal Meningitis among jail prisoners. The best chemoprophylaxis for the protection of contacts is by giving: a) Rifampicin b) Chloramphenicol c) Chloroquine d) Doxycycline e) Penicillin Key: True: a
  • 98. A 5 years old boy developed fever with typical “dew drop rash” over his body. It appeared first on the trunk and spread to arms and legs later; there were no signs of neck stiffness and rigidity. What could be likely diagnosis? a) Small pox b) Measles c) Tanapox d) Meningococcal meningitis e) Chicken pox Key: True: e
  • 99. A school child is diagnosed to have chicken pox. He should be isolated from other school children till: a) The scabs fall off b) Two days after the scabs are formed c) Three days after the fever develops d) Five days after the development of pustules e) Seven days after the development of pustules Key: True: a
  • 100. A mother brought her 4 years old child with complaint of sore throat, difficulty in swallowing and low grade fever. On examination mild erythema and whitish membrane was found on the posterior pharynx. The doctor diagnosed him as a case of Diphtheric Mother gave history of complete course of immunization. In addition to penicillin what would be your line of management? a) Supportive treatment b) Passive immunization c) Active immunization d) Active plus passive immunization e) Active and passive immunization plus Tracheostomy Key: True: b
  • 101. In Pediatric OPD the physician examined a 3 years old child with low grade fever, mild erythema in the throat and whitish membrane on the left side tonsil. The cervical lymph node was palpable. The doctor advised the mother to isolate the child for 7 days from other contacts of less than 5 years old. The most probable diagnosis is a) Pharyngitis b) Tonsillitis c) Diphtheria d) Acute Laryngitis e) Whooping cough Key: True: c
  • 102. A student of nursery class developed mild fever along with irritating cough gradually becoming paroxysmal along with characteristic whoop. What you suggest for how long the student should be isolated from the class? a) I week b) 2 weeks c) 3 weeks d) 4 weeks e) 6 weeks Key: True: d
  • 103. A 30 years old man presented in emergency in POF hospital with complaint of severe headache, fever and vomiting. On examination neck stiffness was found. He has just returned from hajj and gives no history of preceding ailment or injury. The most probable diagnosis is a) Meningitis b) Tetanus c) Brain abscess d) Cerebral Malaria e) Ischemic stroke Key: True: a
  • 104. If you being a field doctor in department of public health, are given a task to visit a low socioeconomic community of a slum and to give report about the immunization status of the community against Tuberculosis. The single most important clue to this immunization is a) Monteux test b) Tuberculosis patients c) BCG scar d) X-ray chest e) Sputum for AFB Key: True: c
  • 105. An epidemiologist was assigned to find out all the cases, both new and old of T.B, in a slum located near Islamabad during year 2007.Prevelance of tuberculosis is confirmed by: a) Mass miniature radiography b) Sputum examination c) Sputum culture d) Tuberculin test e) BCG scar mark Key: True: c
  • 106. A 10 years old boy was brought to a doctor in a hospital with history of moderate fever with shivering and abundant rash on the trunk and buccal mucosa. On examination there were vesicles filled with clear fluid on the trunk and legs. The physician told the mother that child is suffering from a) Herpes b) Chicken pox c) Rubella d) German measles e) Tanapox Key: True: b
  • 107. A pregnant lady reported to ante natal clinic with signs and symptoms of Rubella during 28th week of gestation. She was insisting for the induction of labor because of fear of congenital malformations of fetus. She was told by the doctor that Rubella does not cause major abnormalities of fetus after a) 8th week of pregnancy b) 12th week of pregnancy c) 16th week of pregnancy d) 20th week of pregnancy e) 24th week of pregnancy Key: True: c
  • 108. A 7 years old child presented with sore throat low grade fever rash on face and posterior auricular and cervical lymphadenopathy. The most probable diagnosis is a) Chicken pox b) Small pox c) Measles d) Rubella e) Mumps Key: True: d
  • 109. A 3rd year MBBS student of FJMC presented in ENT OPD with complaint of sore throat, pain of swallowing & low grade fever examination revealed erythema on the pharynx and a whitish membrane on the pharynx extending to the left tonsil. The doctor diagnosed her as a case of diphtheria. What do you suggest minimum isolation period: a) Six daily negative throat and nasal swabs report b) Till the signs & symptoms settle down c) One week course of antibiotic d) For days e) Till complete blood picture becomes normal Key: True: a
  • 110. A mother brought her child with history of paroxysmal cough and restlessness. On examination he showed a loud inspiratory sound and sub-conjuctival haemorrhagic On the basis of clinical presentation what should be the drug of choice: a) Erythromycin b) Ampicillin c) Tetracycline d) Sulphadiazine e) Co- trimoxazole Key: a
  • 111. A 40 year old tuberculosis patient on ATT for the last two months presented to his physician with complaints of tingling, numbness and loss of peripheral sensation. The likely anti tuberculosis drug to have caused these symptoms is: a) Isoniazid b) Rifampicin c) Streptomycin d) Pyrazinamide e) Ethambutol Key: a
  • 112. 10 years old boy presented with high grade fever, chills, aches, cough and generalized weakness. He was diagnosed as a case of influenza. The most dreaded complication is: a) Encephalitis b) Pneumonia c) Toxic shock syndrome d) Reye’s syndrome e) Sub-Conjunctival hemorrhages Key: b
  • 113. A 22 years old married non pregnant woman developed rubella infection. In order to avoid congenital rubella syndrome in her pregnancy she should be given: a) Antibiotics b) Active immunization c) Nonspecific immunization d) Advice to avoid conception for 12 weeks e) Anti viral therapy Key: d
  • 114. A 40 years old man was diagnosed as a case of TB 4 weeks ago. He has been taking ATT for the last 3 weeks. His sputum analysis showed AFB on follow up investigation. Such a case of TB is known as: a) Failure case b) Newer case c) Defaulter d) Transfer out e) Resistant Key: b
  • 115. A primigravida presented in Medical OPD at 39 weeks of gestation with dew drop rash on the body for 1 day she was diagnosed as having chicken pox she was told that her baby is at higher risk of having: a) Low birth weight b) Microcephaly c) Atrophied limbs d) Varicella infections e) Cutaneous scars Key: d
  • 116. A 17 year old boy was brought in emergency department with symptoms of acute encephalopathy. He was admitted in ICU, initial investigation revealed that his liver had undergone fatly degeneration. His father gave history of rash on his body. Most likely he suffered from: a) Measles b) Rubella c) Chicken pox d) Mumps e) Cutaneous diphtheria Key: c
  • 117. After serial sonography it was told to the apparently healthy pregnant woman that her baby is microcephalic and of low birth weight she gave no important medical history of note except mild febrile illness with rapidly disappearing rash in 2nd month of pregnancy. Most likely she suffered from: a) Chicken pox b) German measles c) Measles d) Cutaneous diphtheria e) Malaria Key: b
  • 118. 4 years old girl was having fever, cough with a characteristic whoop. She was diagnosed as a case of whooping cough several antibodies are effective they are important as they: a) Reduce the frequency of spasm b) Control severity of disease c) Shorten the illness d) Control secondary bacterial infection e) Prevent carrier state Key: d
  • 119. A child of 6 presented to school medical officer with complains of fever, malaise and painful swallowing. On examination a diffuse swelling was observed on the sides of the face below and in-front of the ears. The doctor diagnosed him as a case of mumps. What is the most appropriate management for him? a) Analgesics only b) Active and passive immunization c) Passive immunization d) Antibiotics only e) Rest, analgesics and balanced diet Key: e
  • 120. 50 years old Bank officer was told to be suffering from illeocecal Tuberculosis. Nobody suffered tuberculosis in the family, but he still got infected, probably due to: A. Infected milk B. Infected clients C. Contaminated vessel D. Stressful work E. Advancing age Key: a
  • 121. 44 years old man presented with fever, cough, night sweats and weight loss. He is HIV positive , has crackles on both lungs and chest x ray shows bilateral lower lobe infiltrate. All the following are correct except: A. Evaluation should include sputum gram stain and culture B. Sputum should be stained and cultured for M. tuberculosis C. Patient require pneumocystis pneumonia prophylaxis D. Infiltrates in lower lobes exclude diagnosis of TB KEY: D
  • 122. Which of the following groups should not receive live attenuated influenza vaccine: A. Adults with mild to moderate illness B. Healthy children 2- 5 years C. Healthy pregnant D. Penicillin allergic adults Key: c
  • 123. Which statements about Tdap vaccine is false: A. Tdap is contraindicated in pregnancy B. Tdap should replace a single dose of td vaccine for adults who don’t receive a dose of Tdap and require a booster of TD C. Tdap is not licensed for use among adults aged 65 and more D. Tdap is contraindicated in adults allergic to formaldehyde Key: A
  • 124. A 20 month old child sent to your office with mild viral infection . Result of examination was normal except for temperature 37.2 c and clear nasal discharge. She received 2 doses of DTap, OPV and she did not take MMR vaccine. The mother is 20 week pregnant and her brother undergoing chemotherapy for leukemia. Which is more appropriate intervention. A. Scheduale a visit in two weeks for DTaP B. Administer inactivated polio (IPV) and DTaP C. Administer DTaP, opv amd MMR D. Administer DTaP, IPV amd MMR E. Administer DTaP, opv amd MMR after 3 months Key: D
  • 125. Immunization of preschool children with diphtheria toxoid result in: A. Protection against the diphtheria carrier state B. Lifelong immunity against diphtheria C. Detectable antitoxin or immunologic memory for 10 years D. Frequent adverse effects E. Protection against infection with C. diphtheria Key: c
  • 126. What is the recommended interval in months between the administration of whole blood transfusion and MMR vaccine? A. 0 B. 1 C. 3 D. 6 E. 10 Key: D
  • 127. The most common opportunistic infections occuring in HIV patients is: A. Tuberculosis B. Pneumocystis pneumonia C. Amoebic encephalitis D. Fungal pneumonia Key: A
  • 128. A 2 years old boy is brought to ER with sever prostration, temperature 40 c and few petechial lesions around the ankle. A gram stian of blood showed gram negative diplococci , what is the case fatality rate of this disease? A. 5 – 15 % B. 20 – 30 % C. 40 – 50 % D. More than 50% Key: A
  • 129. CONTINUE Compare to general population the risk of developing infection in household contacts is: A. the same B. 10 – 20 time more C. 50 -100 time more D. 200 -400 time more E. 500 – 800 time more key: E
  • 130. CONTINUE The child had been to child day care center, in addition to close surveillance, which of the following is most appropriate intervention? A. No further action B. Vaccination of children only C. Vaccination of children and adults D. Antibiotic prophylaxis for children only E. Antibiotic prophylaxis for children and adults Key: E
  • 131. A 7 years old girl brought to your office because of a rash that appeared 3 days ago. Her temp. was 37.2 c and her face has intense rash with a slapped cheek appearance. The most likely etiological factor is: A. Adenovirus B. Rotavirus C. Parvovirus D. Coxsackie virus E. Echovirus Key: c
  • 132. To which patient MMR would be safe to administer? A. A 15 months old HIV infected child with CD4 cell count 700 B. A 25 years old pregnant woman C. A 12 years old asthmatic on 20 mg of oral prednisone daily for the last 20 days D. A 18 years old leukemia in remission whose chemotherapy was terminated 1 month ago E. A 17 years old with life threatening anaphylactic reaction to egg Key: A
  • 133. A 19 years old colleague student complaining of sever coughing spells for the last 4 days following initial symptoms of coryza and malaise. She is afebrile. In weekends she baby sits a 10 month and 2 years old children, in term of controlling contacts , What is the most important etiological factor to be included in D.D? A. Streptococcus pneumoniae B. Mycoplasma pneumoniae C. Bordetella pertussis D. Influenza virus E. Legionella Key: c
  • 134. Which of the following infections is transmitted mainly from person to person? A. California encephalitis B. St. louis encephalitis C. West Nile viral encephalitis D. Meningococcal meningitis E. Eastern equine encephalitis Key: D
  • 135. In investigation about influenza outbreak, you found out that a large number of persons who developed mild symptoms, have been vaccinated with trivalent vaccine containing the appropriate strain for the current year. The isolated strain is the same strain in the trivalent vaccine. What is the most likely explanation? A. Vaccine failure B. Antigenic drift C. Antigenic shift D. Herd immunity E. Incomplete immunity due to rhinovirus infection Key: B
  • 136. A 5 years old preschool child presented with fever, malaise and vesicular rash that started 24 h ago. He has a sibling 3 months old and pregnant mother 38 week both are susceptible. What is the most common complication? A. Pneumonia B. Reyes syndrome C. Encephalitis D. Orchitis E. Thrombocytopenia Key: A
  • 137. CONTINUE What is the most appropriate management for contacts? A. Observation only for all contacts B. Vaccination for the mother, sibling and class mates C. Immune globulins for mother, sibling and class mates D. Immune globulins for mother and vaccination for sibling and class mates E. Immune globulins for mother and sibling and vaccination for class mates Key: E
  • 138. Under which condition should chemoprophylaxis for influenza be considered? A. All nursing home residents and unvaccinated staff during an influenza A outbreak B. All nursing home residents and unvaccinated staff during an influenza B outbreak C. Only unvaccinated nursing home residents and staff during an influenza A outbreak D. Only unvaccinated nursing home residents and staff during an influenza B outbreak E. All nursing home residents and staff during an influenza B outbreak Key: A
  • 139. For which patient pneumococcal vaccine PPV23 is not beneficial? A. A 15 month old HIV infected child B. A 20 years old about to undergo splenectomy for ITP C. A 70 years old healthy female D. A 5 years old child with sickle cell disease E. A 10 years old boy with nephrotic syndrome who received the vaccine 5 years ago. Key: A
  • 140. A 32 years old HIV infected patient, has 5 mm induration after tuberculin test. His chest x ray is normal, he is currently on protease inhibitors antiretroviral treatment. He did not receive anti tuberculosis treatment nor he had been in contact with tuberculosis patient. What is the most appropriate intervention? A. No preventive therapy for TB B. Isoniazid for 9 months C. Rifampicin for 9 months D. Rifampicin and pyrazinamide for 2 months E. Streptomycin for 6 months Key: B
  • 141. HIV infected patients have high risk of developing active TB resistant to: A. Isoniazid B. Rifampicin C. Streptomycin D. Pyrazinamide E. Ethambutole Key: A
  • 142. Four drug therapy is recommended as initial approach to HIV infected active TB patient: A. Always B. When multidrug resistant TB exceed 4% in community C. When the patient had anti tuberculosis treatment before D. When the patient has contact with multidrug resistant TB E. When CD4 count is under 200 Key: A
  • 143. A 30 years old farmer suffering from fever, coughing, night sweats and malaise. He received BCG when he was child and his skin test is 15 mm, the most likely diagnosis; A. Influenza B. Brucellosis C. Aspergillosis D. Mycobacterium tuberculosis E. Mycobacterium bovis Key: E
  • 144. A 4 years old girl suffering from sore throat, fever, hoarseness and drolling. What is the most likely diagnosis? A. Poliovirus B. C. Tetani C. C. Diphtheria D. H. Influenza E. Streptococci Key: D